Spine Flashcards

1
Q

The images shown depict the standing posteroanterior and lateral radiographs of a 64-year-old woman with adult degenerative scoliosis who underwent a decompressive laminectomy and fusion from L2-5. She now reports progressive lower back pain and a feeling of being shifted forward and to the left side. If surgical intervention is considered, correction of which of the following is most likely to improve her health-related quality of life outcomes?
Answers:
A. Sacral slope
B. Coronal balance
C. Sagittal balance
D. Coronal cobb angle
E. Pelvic Tilt

A

Sagittal balance

Discussion:
Correction of the patient’s positive sagittal balance is most likely to improve her health-related
quality of life outcomes. Severity of symptoms increases in a linear fashion with progressive
positive sagittal balance. This correlation was not noted for coronal alignment. Relative kyphosis is
poorly tolerated in the lumbar spine. Pelvic tilt and sacral slope are measurements to be
considered when planning to correct a sagittal balance issue; however, these do not answer the
question asked. The sacral slope is the angle between the horizontal plane and the cephalad
endplate of the S1 vertebra. Cobb angle is a measurement of the degree of scoliosis in the coronal
view.
References:
Glassman SD, Bridwell K, Dimar JR, Horton W, Berven S, Schwab F. The impact of positive
sagittal balance in adult spinal deformity. Spine (Phila Pa 1976). 2005 Sep 15;30(18):2024-2029.
Pubmed Web link: https://journals-lww-com.geihsl.idm.oclc.org/spinejournal/Fulltext/2005/09150
/The_Impact_of_Positive_Sagittal_Balance_in_Adult.5.aspx
Glassman SD, Berven S, Bridwell K, Horton W, Dimar JR. Correlation of radiographic parameters
and clinical symptoms in adult scoliosis. Spine (Phila Pa 1976). 2005 Mar 15;30(6):682-688.
Pubmed Web link: https://pubmed.ncbi.nlm.nih.gov/15770185/

How well did you know this?
1
Not at all
2
3
4
5
Perfectly
2
Q

What is the most common cause of symptoms from a spinal dural arteriovenous fistula?
Answers:
A. Venous hypertension
B. Spinal epidural hematoma
C. Spinal subarachnoid hemorrhage
D. Intraparenchymal spinal cord hemorrhage
E. Neurovascular compression

A

Venous hypertension

Discussion:
Spinal arteriovenous malformations are classically divided into 4 types:
1) Dural root sleeve arteriovenous-fistula
2) Intramedullary glomus AVM
3) Intramedullary/extramedullary AVM + paraspinal juvenile AVM
4) Intradural perimedullary arteriovenous-fistula
Symptoms from spinal dural AV-fistulas typically arise from venous hypertensive myelopathy.
Venous congestion causes outflow obstruction from the spinal cord. This subsequently causes
spinal cord edema with myelopathic signs and symptoms. Spinal dural AV-fistulas are treated by
either super-selective endovascular embolization or microsurgical ligation of the shunting zone.
References:
Sucuoğlu H, Aktürk A. Spinal dural arteriovenous fistula: A rare cause of progressive myelopathy
and bladder and bowel dysfunction. Turk J Phys Med Rehabil. 2020 May 18;66(2):219-222. doi:
10.5606/tftrd.2020.3732. PMID: 32760901; PMCID: PMC7401687
Pubmed Web link: https://pubmed.ncbi.nlm.nih.gov/32760901/
Jablawi F, Mull M. The clinical value of venous drainage in patients with spinal dural arteriovenous
fistula. J Neurol Sci. 2019 Feb 15;397:50-54. doi: 10.1016/j.jns.2018.12.024. Epub 2018 Dec 18.
PMID: 30590341
Pubmed Web link: https://pubmed.ncbi.nlm.nih.gov/30590341/

How well did you know this?
1
Not at all
2
3
4
5
Perfectly
3
Q

Which of the following methods is used to calculate pelvic incidence?
Answers:
A. Angle between a line vertical from the mid femoral point and a line to the posterior cranial endplate of S1 from the mid femoral point
B. Angle between a line drawn from the midpoint of the femoral heads to the midpoint of the superior endplate of S1 and a line perpendicular vertical from the midpoint of the femoral heads
C. Angle between the line perpendicular to the middle of the cranial sacral end plate and the line joining the middle of the cranial sacral endplate to the center of the midpoint of the femoral heads
D. The angle between the cranial end plate of L1 and caudal endplate of L5
E. Angle between the horizontal line and the cranial sacral end plate

A

Angle between the line perpendicular to the middle of the cranial sacral end plate and the line joining the middle of the cranial sacral endplate to the center of the midpoint of the femoral heads

Discussion:
Pelvic incidence (normally approximately 50 degrees) is the angle between the line perpendicular
to the middle of the cranial sacral end plate and the line joining the middle of the cranial sacral
endplate to the center of the midpoint of the femoral heads. The angle between the cranial
endplate of L1 and caudal endplate of L5 is one way to describe the lumbar lordosis (normally
20-40 degrees). Some suggest incorporating the cranial endplate of S1 to account for the lordosis
achieved at the L5-S1 space. Pelvic tilt (normally 10-25 degrees) is the angle between a line
drawn from the midpoint of the femoral heads to the midpoint of the superior endplate of S1 and a
line perpendicular vertical from the midpoint of the femoral heads. The angle between the
horizontal line and the cranial sacral end plate is the sacral slope (normally 36-42 degrees). The
angle between a line vertical from the mid femoral point and a line to the posterior cranial endplate
of S1 from the mid femoral point is incorrect; the line to the midpoint of the cranial endplate of S1
would be pelvic tilt.
References:
Vialle R, Levassor N, Rillardon L, et al. Radiographic analysis of the sagittal alignment and balance
of the spine in asymptomatic subjects. J Bone Joint Surg Am. 2005 Feb;87(2):260-7.
Pubmed Web link: https://journals-lww-com.geihsl.idm.oclc.org/jbjsjournal/Fulltext/2005/02000
/Radiographic_Analysis_of_the_Sagittal_Alignment.4.aspx
Schwab FJ, Blondel B, Bess S, et al. Radiographical spinopelvic parameters and disability in the
setting of adult spinal deformity: a prospective multicenter analysis. Spine (Phila Pa 1976).2013
Jun 1;38(13):E803-812

How well did you know this?
1
Not at all
2
3
4
5
Perfectly
4
Q

The nerve roots in a lipomyelomeningocele typically lie in which of the following locations?
Answers:
A. Within the fat
B. Dorsal and lateral
C. Ventral and lateral
D. Dorsal and medial
E. Ventral and medial

A

Ventral and lateral

iscussion:
The type of spinal lipoma described, a lipomyelomeningocele, is where part of the distal conus
extends into the extraspinal compartment, dragging with it a small collar of dural sac. The basic
structure is that of either a transitional or a dorsal lipoma. Dorsal lipoma is found entirely on the
dorsal surface of the lumbar spinal cord, always sparing the distal conus. Junctional demarcation (the fusion line) is between the lipoma, cord, and pia and can be traced in an oval track separating the fat from the dorsal root entry zone (DREZ) and the dorsal nerve roots laterally. A transitional lipoma is similar to a dorsal lipoma but involves the conus; neural tissue is always ventral to this interface (i.e., on the side of the nerve roots exit), and the DREZ and the nerve roots are predictably localizable lateral and ventral to the fusion line and therefore do not course through the fat.
References:
Albright, A. L., Adelson, P. D., & Pollack, I. F. (2008). Principles and practice of pediatric
neurosurgery. New York: Thieme.
Web link: https://www.wolterskluwer.com/en/solutions/ovid/principles-and-practice-of-pediatricneurosurgery-13926
Pang D, Zovickian J, Wong ST, Hou YJ, Moes GS. Surgical treatment of complex spinal cord
lipomas. Childs Nerv Syst. 2013 Sep;29(9):1485-513. doi: 10.1007/s00381-013-2187-4. PMID:
24013320.
Pubmed Web link: https://pubmed.ncbi.nlm.nih.gov/24013320/

How well did you know this?
1
Not at all
2
3
4
5
Perfectly
5
Q

Which of the following treatment modalities is associated with the greatest risk of morbidity and mortality for patients older than 75 years of age who have type II odontoid fractures?
Answers:
A. rigid orthosis/hard collar
B. Posterior C1-C2 fusion
C. Odontoid screw fixation
D. Halo vest immobilization
E. Soft collar

A

Halo vest immobilization

Discussion:
Placement of elderly patients with type 2 odontoid fractures in halo-vest immobilization is
associated with an increased risk of morbidity and mortality at 30 days after injury compared to
either operative intervention or rigid cervical orthosis. A large North American study showed that
surgical treatment of type II odontoid fracture in an elderly population did not negatively impact
survival, even after adjusting for age, sex, and comorbidities. The data suggest a significant 30-
day survival advantage and a trend toward improved longer-term survival for operatively treated
over non-operatively treated patients.
References:
Tashjian RZ, Majercik S, Biffl WL, Palumbo MA, Cioffi WG. Halo-vest immobilization increases
early morbidity and mortality in elderly odontoid fractures. J Trauma. 2006 Jan;60(1):199-203. doi:
10.1097/01.ta.0000197426.72261.17. PMID: 16456456.
Pubmed Web link: https://pubmed.ncbi.nlm.nih.gov/16456456/
Chapman J, Smith JS, Kopjar B, Vaccaro AR, Arnold P, Shaffrey CI, Fehlings MG. The AOSpine
North America Geriatric Odontoid Fracture Mortality Study: a retrospective review of mortality
outcomes for operative versus nonoperative treatment of 322 patients with long-term follow-up.
Spine (Phila Pa 1976). 2013 Jun 1;38(13):1098-104. doi: 10.1097/BRS.0b013e318286f0cf. PMID:
23354104; PMCID: PMC3678887.
Pubmed Web link: https://pubmed.ncbi.nlm.nih.gov/23354104/

How well did you know this?
1
Not at all
2
3
4
5
Perfectly
6
Q

Which of the following materials has been supported with significant clinical data to have equal or superior fusion rates when compared with those of an iliac crest autograft?
Answers:
A. Demineralized bone matrix (DBM)
B. Synthetic bone graft
C. Bone morphogenic protein (rhBMP-2)
D. Platelet rich gels
E. Femoral head allograft

A

Bone morphogenic protein (rhBMP-2)

Discussion:
Bone Morphogenic Protein, or BMP, has been shown to have equal or superior fusion rates when
compared with those of an iliac crest autograft. The morbidity associated with iliac crest harvesting
has led to efforts to assess bone graft alternatives. BMP is FDA-approved for use in anterior
lumbar interbody fusion, and has been shown to yield fusion rates of 95% to 100%. All other
answer choices (synthetic bone grafts, platelet gels, allograft, and DBM) have been shown to have
similar or worse fusion rates compared to fusion rates of iliac crest autograft.
Synthetic bone grafts with beta tricalcium phosphate, for example, have been shown to only act as
a scaffold, showing fusion rates as low as 0% when used alone compared to 67% when autograft
was used in conjunction.
Platelet gels contain platelet rich plasma and are osteoconductive. Iliac crest autograft fusion rates
alone have been shown to be 91%, compared to 62% when combined with platelet gels. Another
study showed a 25% non-union rate in the platelet gel group vs 17% non-union rate in the iliac
crest group.
DBM has been shown to have a fusion rate of 83% compared to iliac crest fusion rate of 85%, with
no statistical difference, although significant clinical data is lacking.
Allografts, such as from the femoral head, have been shown to have a fusion rate of 70%
compared to 92% fusion rates with autologous iliac bone. Some studies show similar fusion rates,
but significant clinical data is lacking.
References:
Rihn J, Kirkpatrick K, Albert TJ. Graft options in posterolateral and posterior interbody lumbar
fusion. SPINE 2010: 35(17) 1629-1639
Pubmed Web link: https://journals-lww-com.geihsl.idm.oclc.org/spinejournal/Fulltext/2010/08010
/Graft_Options_in_Posterolateral_and_Posterior.9.aspx
Kaiser MG, Groff MW, Watters WC 3rd, Ghogawala Z, Mummaneni PV, Dailey AT, et al. Guideline
update for the performance of fusion procedures for degenerative disease of the lumbar spine.
Part 16: bone graft extenders and substitutes as an adjunct for lumbar fusion. J Neurosurg Spine.
2014 Jul;21(1):106-32

How well did you know this?
1
Not at all
2
3
4
5
Perfectly
7
Q

The pain afferents from the skin have their first synapse in which of the following?
Answers:
A. Posterolateral Nucleus of Thalamus
B. Geniculate Ganglion
C. Spinal Nucleus
D. Stellate Ganglion
E. Dorsal Root Ganglion

A

Spinal Nucleus

Discussion:
The correct answer is the Spinal Nucleus of the Trigeminal Nerve in the Pons/Medulla.
In the pain pathway, first order pseudo-unipolar neurons are located in peripheral ganglia such as
Dorsal Root Ganglia (DRG) for spinal nerves, Trigeminal Ganglion and the Geniculate Ganglion for
the Facial (VII) nerve. Second order neurons are located in the dorsal horn (Rexed Laminae III-V)
of the spinal cord. Lamina III-IV are referred to as the Nucleus Proprius, the location of the first
synapse in the pain pathway within the spinal cord. This extends more cephalad into the brainstem
in the form of the Spinal Nucleus of Trigeminal Nerve. The Pars Caudalis subdivision specifically is
responsible for the transmission of pain and temperature relayed from Cranial nerves V, VII, IX,
and X, and project through the spinal tract to the thalamus. Interneurons in the Substantia
Gelatinosa of Rolando (SGR) - also referred to as Rexed Lamina II- correlate with their role in pain
modulation. 3rd order neurons are in the Thalamus (posterolateral nucleus) with projections to the
somatosensory cortex. Alternatively, some second order neurons synapse in the Reticular
formation instead (indirect route), with reticulo-thalamic tracts projecting to the thalamus instead.
The Stellate Ganglion (cervico-thoracic ganglion) is a sympathetic chain ganglion with efferent preganglionic (2nd order neuron) fibers originating from the Intermediolateral horn neurons (T1-L3),
and synapsing with post-ganglionic fibers (3rd order neuron) that continue around the carotid and
vertebral arteries.
References:
Carpenter MB. Core Text of Neuroanatomy. 4th ed. Baltimore: Williams and Wilkins; 1991: 86-87.
Pain and Disability: Clinical, Behavioral, and Public Policy Perspectives. Institute of Medicine (US)
Committee on Pain, Disability, and Chronic Illness Behavior; Osterweis M, Kleinman A, Mechanic
D, editors. Washington (DC): National Academies Press (US); 1987.
Pubmed Web link: https://www.ncbi.nlm.nih.gov/books/NBK219252/

How well did you know this?
1
Not at all
2
3
4
5
Perfectly
8
Q

A 34-year-old man presents with a progressive thoracic myelopathy. The MR image shown depicts a mural nodule with an associated cyst. The mural nodule enhances with gadolinium. Preoperative hematocrit is 51%. The most appropriate initial management step is:
Answers:
A. Observation and 6 month follow-up MRI
B. Biopsy and stereotactic radiosurgery
C. Lumbar puncture
D. Abdominal images and catecholamine assessment
E. External beam radiotherapy and physical therapy

A

Abdominal images and catecholamine assessment

Discussion:
Approximately 25-33% of patients harboring a hemangioblastoma have Von Hippel-Lindau (VHL)
disease. In patients with suspected VHL, diagnostic workup of the neuraxis and retina should be
undertaken to look for additional hemangioblastomas. VHL carries a significant risk of
development of both renal cell carcinoma and pheochromocytoma. Due to the risk of sudden
catecholamine release from surgery, VHL patients should have a screening abdominal ultrasound
or CT scan. The MR imaging in this case is not typical for multiple sclerosis or other demyelinating
diseases, therefore MRI of the brain and spinal tap are not indicated. Hemangioblastomas of the
spinal cord are benign tumors with a distinct border between tumor and spinal cord. They may be
cured by total surgical excision. Therefore, external beam radiotherapy and/or conservative
management are not indicated. Open biopsy is an option, but once the diagnosis of
hemangioblastoma is confirmed, gross total excision should be undertaken.
References:
Korf BR. The phakomatoses. Neuroimaging Clin N Am. 2004 May;14(2):139-48, vii. doi:
10.1016/j.nic.2004.03.008. PMID: 15182812.
Pubmed Web link: https://pubmed.ncbi.nlm.nih.gov/15182812/
Maher ER, Kaelin WG Jr. von Hippel-Lindau disease. Medicine (Baltimore). 1997
Nov;76(6):381-91.

How well did you know this?
1
Not at all
2
3
4
5
Perfectly
9
Q

A 45-year-old man with an eight-week history of persistent, subacute, lower back pain is prescribed light duty and rest. On follow-up, the pain persists. Neurological examination and MR imaging show no abnormalities. Which of the following is the most appropriate next step in management?
Answers:
A. Interventional pain management for consideration of facet injections, epidural steroid injections, & facet rhizotomies
B. Acute inpatient rehabilitation with 2 weeks focus on range of motion exercises, back and core muscle strengthening exercises, massage therapy and aquatic therapies
C. Physical therapy, massage therapy, gait retraining and narcotic prescription for management of low back pain
D. Interventional pain management and medical pain management referrals for consideration of lumbar injections and medication management to avoid narcotics
E. Intensive interdisciplinary rehabilitation, exercise therapy, acupuncture, massage therapy, spinal manipulation, yoga, & cognitive-behavioral therapy

A

Intensive interdisciplinary rehabilitation, exercise therapy, acupuncture, massage therapy, spinal manipulation, yoga, & cognitive-behavioral therapy

Discussion:
The most appropriate next step in management is intensive interdisciplinary rehabilitation, exercise
therapy, acupuncture, massage therapy, spinal manipulation, yoga, and cognitive-behavioral
therapy. These interventions would not require an inpatient acute rehab stay. This patient has
subacute low back pain and has not maximized his conservative management options. Prior to
considering any invasive procedures, including interventional pain management injections, and
prior to considering the addition of narcotic medications, conservative, non-invasive therapies must
be attempted. Moreover, it would be difficult for any targeted injections to be prescribed to this
patient with a reasonable degree of proposed symptomatic relief without significant MRI findings.
References:
Chou R, et al. Diagnosis and treatment of low back pain: a joint clinical practice guideline from the
American College of Physicians and the American Pain Society. Ann Intern Med. 2007 Oct 2; 147
(7): 478-491.
Pubmed Web link: https://www.acpjournals.org/doi/full/10.7326
/0003-4819-147-7-200710020-00006?rfr_dat=cr_pub++0pubmed&url_ver=Z39.88-2003&
rfr_id=ori%3Arid%3Acrossref.org

How well did you know this?
1
Not at all
2
3
4
5
Perfectly
10
Q

A 19-year-old man is in the ICU two days after experiencing an American Spinal Injury Association (ASIA) Impairment Scale A cervical spinal cord injury. Surgical intervention is not planned in the near future. Which of the following is the most appropriate treatment for deep venous thrombosis (DVT) prophylaxis at this time?
Answers:
A. Mechanical prophylaxis now and pharmacological prophylaxis after 72 hours after injury
B. Surveillance lower extremity ultrasounds only
C. Mechanical prophylaxis only
D. Pharmacological prophylaxis and mechanical prophylaxis within 72 hours of injury
E. There is no evidence for need for DVT prophylaxis in this population

A

Pharmacological prophylaxis and mechanical prophylaxis within 72 hours of injury

Discussion:
Multiple literature reviews and current clinical guidelines (including current CNS spinal cord injury
guidelines) support early deep venous thrombosis (DVT) prophylaxis in patients with acute spinal
cord injury. This patient population is particularly vulnerable to thromboembolic events due to direct
tissue trauma, associated traumatic injuries, and functional immobility secondary to spinal cord
injury. Early thromboprophylaxis is recommended and may include mechanical prophylaxis with
lower extremity compression devices as well as pharmacological agents such as heparin or
enoxaparin. A 2017 clinical practice guideline by Fehlings et al., recommends using either low
dose unfractionated heparin or low molecular weight heparin within 72 hours of injury.
References:
Fehlings MG, Tetreault LA, Aarabi B, Anderson P, Arnold PM, Brodke DS, Burns AS, Chiba K,
Dettori JR, Furlan JC, Hawryluk G, Holly LT, Howley S, Jeji T, Kalsi-Ryan S, Kotter M, Kurpad S,
Kwon BK, Marino RJ, Martin AR, Massicotte E, Merli G, Middleton JW, Nakashima H, Nagoshi N,
Palmieri K, Singh A, Skelly AC, Tsai EC, Vaccaro A, Wilson JR, Yee A, Harrop JS. A Clinical
Practice Guideline for the Management of Patients With Acute Spinal Cord Injury:
Recommendations on the Type and Timing of Anticoagulant Thromboprophylaxis. Global Spine J.
2017 Sep;7(3 Suppl):212S-220S. doi: 10.1177/2192568217702107. Epub 2017 Sep 5. PMID:
29164026; PMCID: PMC5684841.
Pubmed Web link: https://pubmed.ncbi.nlm.nih.gov/29164026/
Arnold PM, Harrop JS, Merli G, Tetreault LG, Kwon BK, Casha S, Palmieri K, Wilson JR, Fehlings
MG, Holmer HK, Norvell DC. Efficacy, Safety, and Timing of Anticoagulant Thromboprophylaxis for
the Prevention of Venous Thromboembolism in Patients With Acute Spinal Cord Injury: A
Systematic Review. Global Spine J. 2017 Sep;7(3 Suppl):138S-150S. doi:
10.1177/2192568217703665. Epub 2017 Sep 5. PMID: 29164021; PMCID: PMC5684847.
Pubmed Web link: https://pubmed.ncbi.nlm.nih.gov/29164021/

How well did you know this?
1
Not at all
2
3
4
5
Perfectly
11
Q

Which of the following lateral plain film measurements of the cervical spine is used to make the diagnosis of atlanto-occipital dislocation?
Answers:
A. Odontoid angular ratio
B. Jefferson’s ratio
C. Condylar gap
D. C1-2 interspinous gap
E. Rule of Spence

A

Condylar gap

Discussion:
Atlanto-occipital dislocation is often the result of a high velocity trauma resulting in a ligamentous
disarticulation of the cranium relative to the spinal column. It can easily be missed on radiographic
imaging. While it is more reliable to identify AO-dislocation on CT scans, it is important to be aware
of the plain film radiographic measurements used for diagnosis. There are some limitations to the
accuracy of interpretation of these diagnostic measurements on lateral plain films as opposed to
CT. A fairly reliable method, however, is to examine the degree of distraction between the occipital
condyle and the C1 lateral mass. This measurement is called the condylar gap (AKA condyle-C1
interval). The occipital condyle and C1 lateral mass should be closely apposed. Visible distraction
on lateral plain films should be confirmed with a CT scan. In adults, a condylar gap of >3mm on CT
scan is considered abnormal. It is important to assess the coronal films, as there is often some
element of asymmetry of the O-C1 joints in AO-dislocation. These findings can further be
confirmed with an MRI scan, which would reveal STIR signal hyperintensity within the O-C1 joints.
Widening of the C1-2 interspinous space may be appreciated radiographically when there is
significant injury to the atlanto-axial complex. However, it is not indicative of atlanto-occipital
dislocation.
The Rule of Spence is applied when assessing the integrity of the transverse cruciate ligament, not
the atlanto-axial ligamentous complex. On an open-mouth odontoid view, >7mm overhang of the
C1 lateral mass relative to C2 is indicative of an injury to the transverse cruciate ligament.
Jefferson’s ratio and the odontoid angular ratio are not reflective of applied measurements
described in the literature.
References:
Pang D, Nemzek WR, Zovickian J. Atlanto-occipital dislocation–part 2: The clinical use of
(occipital) condyle-C1 interval, comparison with other diagnostic methods, and the manifestation,
management, and outcome of atlanto-occipital dislocation in children. Neurosurgery. 2007
Nov;61(5):995-1015; discussion 1015. doi: 10.1227/01.neu.0000303196.87672.78. PMID:
18091277
Pubmed Web link: https://pubmed.ncbi.nlm.nih.gov/18091277/
Dziurzynski K, Anderson PA, Bean DB, Choi J, Leverson GE, Marin RL, Resnick DK. A blinded
assessment of radiographic criteria for atlanto-occipital dislocation. Spine (Phila Pa 1976). 2005
Jun 15;30(12):1427-32. doi: 10.1097/01.brs.0000166524.88394.b3. PMID: 15959373
Pubmed Web link: https://pubmed.ncbi.nlm.nih.gov/15959373/

How well did you know this?
1
Not at all
2
3
4
5
Perfectly
12
Q

A type 1 odontoid fracture most likely indicates an avulsion injury of which of the following structures?
Answers:
A. Transverse ligament
B. Ligamentum flavum
C. Posterior longitudinal ligament
D. Tectorial membrane
E. Apical ligament

A

Apical ligament

Discussion:
Type 1 odontoid fractures are fractures of the rostral tip of the odontoid process that may be
associated with avulsion injury to the apical ligament. The apical ligament attaches from the top of
the odontoid process to the basion. Disruption of this attachment can result in craniocervical
instability. The transverse ligament attaches the odontoid process to the anterior arch of C1.
Ligamentum flavum attaches adjacent lamina to each other. The tectorial membrane (otherwise
known as the tectorial ligament) is an extension of the posterior longitudinal ligament the connects
the posterior aspect of the odontoid and C2 vertebral body to the basion and C3 vertebral body.
The posterior longitudinal ligament connects the posterior surface of adjacent vertebral bodies and
provides significant biomechanical support of a motion segment.
References:
Tubbs RS, Kelly DR, Humphrey ER, Chua GD, Shoja MM, Salter EG, Acakpo-Satchivi L, Wellons
JC 3rd, Blount JP, Oakes WJ. The tectorial membrane: anatomical, biomechanical, and histological
analysis. Clin Anat. 2007 May;20(4):382-6. doi: 10.1002/ca.20334. PMID: 16617439.
Pubmed Web link: https://pubmed.ncbi.nlm.nih.gov/16617439/
Carvalho AD, Figueiredo J, Schroeder GD, Vaccaro AR, Rodrigues-Pinto R. Odontoid Fractures: A
Critical Review of Current Management and Future Directions. Clin Spine Surg. 2019
Oct;32(8):313-323. doi: 10.1097/BSD.0000000000000872. PMID: 31464693.
Pubmed Web link: https://pubmed.ncbi.nlm.nih.gov/31464693

How well did you know this?
1
Not at all
2
3
4
5
Perfectly
13
Q

A 66-year-old man is evaluated because of leg weakness, unsteadiness, and inability to walk effectively. Examination shows strength in all lower extremity muscle groups is 5/5 and he has normal sensation to light touch in both lower extremities. He is slow in getting up from a seated position and has an unsteady, wide-based gait. An MR image of the lumbar spine is shown. Which
of the following is the most appropriate next step in management?
Answers:
A. MRI of the cervical spine without gadolinium
B. No further imaging, physical therapy, gait retraining & close follow up
C. MRI of the brain, cervical, and thoracic spine without and with gadolinium
D. MRI of the lumbar spine with gadolinium
E. Repeat MRI of the lumbar spine without gadolinium in 6 months

A

MRI of the cervical spine without gadolinium

Discussion:
The most appropriate next step in management would be to obtain an MRI of the cervical spine.
The patient’s signs and symptoms suggest myelopathy. An MRI of the cervical spine without
gadolinium would be the best test to assess for cervical spondylotic myelopathy. This may reveal
cervical spondylosis, multilevel central and foraminal stenosis with spinal cord compression and
possible findings of T2 changes suggestive of cord edema or myelomalacia. The MRI of the
lumbar spine shown reveals moderate lumbar spondylosis without any significant central canal
stenosis or cauda equina nerve root compression. The findings in this patient may also be from
pathologies in the thoracic spine, but these are statistically less common. Conservative
management with physical therapy, close monitoring and repeat MRI in 6 months of the lumbar
spine would not be appropriate.
References:
Salvi FJ, Jones JC, Weigert BJ. The assessment of cervical myelopathy. Spine J. 2006 NovDec;6(6 Supplement):182S-189S.
Harrop JS, Naroji S, Maltenfort M, et al. Cervical myelopathy: a clinical and radiographic evaluation
and correlation to cervical spondylotic myelopathy. Spine (Phila Pa 1976). 2010 Feb 10. Epub
ahead of print

How well did you know this?
1
Not at all
2
3
4
5
Perfectly
14
Q

A 75-year-old woman with severe back pain has no abnormalities on neurological examination. X-ray films are shown. She was never diagnosed with scoliosis during her childhood or adolescence. Which of the following is the most likely diagnosis?
Answers:
A. Congenital scoliosis
B. Neuromuscular scoliosis
C. Idiopathic scoliosis
D. Scheuermann’s kyphosis
E. Degenerative scoliosis

A

Degenerative scoliosis

Discussion:
The patient’s x-rays show a thoracolumbar/lumbar scoliosis and severe kyphosis. Without a prior
diagnosis of idiopathic scoliosis, and without any congenital malformations – such as tethered
cord, Klippel-Feil, etc. – this patient is best categorized as a degenerative scoliosis.
Neuromuscular scoliosis is mainly a pediatric diagnosis in children with cerebral palsy and other
neuromuscular disorders. Scheuermann’s Kyphosis is a distinct pattern of mid-thoracic kyphosis
with wedging of multiple vertebrae, often without a scoliosis. For this to be an “idiopathic” scoliosis,
she would have to have a diagnosis of AIS or an earlier diagnosis as a young adult of adult
idiopathic scoliosis (AdIS). Of note, these are AP x-rays, and scoliosis films are often viewed in the
PA (posterior-anterior) manner, where left is left and right is right, such that the heart is on the left
side of the screen. Also of note, it is crucial to assess the lumbosacral fractional curve in these
patients, which is the coronal Cobb angle from L4 to the sacrum, (though can be measured from
L3 or L5). This patient has a left concavity lumbosacral fractional curve. Although this particular
patient has no radicular symptoms, patients are often symptomatic on the concavity of the
lumbosacral fractional curve due to foraminal stenosis causing lumbar radiculopathy. It is important
to note that globally (assessing head to pelvis), she is maligned in the sagittal and coronal planes.
Her SVA is approximately 7-9cm, and coronally her head centered to the left of her pelvis.
References:
Oskouian RJ Jr., Shaffrey CI. Degenerative lumbar scoliosis. Neurosurg Clin N Am. 2006
Jul;17(3):299-315, vii.
Pubmed Web link: https://pubmed-ncbi-nlm-nih-gov.proxy.library.vanderbilt.edu/16876030/
Birknes JK, White AP, Albert TJ, Shaffrey CI, Harrop JS. Adult degenerative scoliosis: a review.
Neurosurgery. 2008 Sep;63(3 Suppl):94-103. PMID: 18812938
Pubmed Web link: https://pubmed-ncbi-nlm-nih-gov.proxy.library.vanderbilt.edu/188129

How well did you know this?
1
Not at all
2
3
4
5
Perfectly
15
Q

A ruptured intervertebral disc at C6-7 most commonly causes:
Answers:
A. Weakness in elbow extension
B. Diminished brachioradialis reflex
C. Diminished finger-jerk reflex
D. Hypoesthesia in fingers 4 and 5
E. Weakness in the hand intrinsic muscles

A

Weakness in elbow extension

Discussion:
A C6-7 disc herniation compresses the C7 nerve root which will cause weakness in elbow
extension, diminished triceps reflex and hypoesthesia in digits 2 and 3. Hand intrinsic muscles,
sensation in digits 4 and 5, and finger-jerk reflex are supplied by C8. Brachioradialis is mainly
supplied by C6 (Table).
References:

.
Cervical disk
herniation
C4-5 C5-6 C6-7 C7-T1
Compressed nerve root C5 C6 C7 C8
Diminished reflex Deltoid & pectoralis Biceps & brachioradialis Triceps Finger-jerk
Motor weakness Deltoid Elbow flexion Elbow
extension
Hand
intrinsics
Hypoesthesia and
paresthesia Shoulder
Upper arm,
thumb, radial
forearm
Fingers 2 &
3,
all
fingertips
Fingers 4 &
5

Sharrak S, Al Khalili Y. Cervical Disc Herniation. [Updated 2021 Sep 3]. In: StatPearls [Internet].
Treasure Island (FL): StatPearls Publishing; 2021 Jan-. Available from:
https://www.ncbi.nlm.nih.gov/books/NBK546618/
Web link: https://www.ncbi.nlm.nih.gov/books/NBK546618/
Greenberg, M. S. (2020). Handbook of Neurosurgery. New York: Thieme Medical Publishers
Pubmed Web link: https://www.thieme.com/books-main/neurology/product/5411-handbook-ofneurosurgery

How well did you know this?
1
Not at all
2
3
4
5
Perfectly
16
Q

A 38-year-old man is evaluated because of a six-month history of lower back pain that improves with exercise but worsens with rest. He reports having stiffness and pain in the morning. Medical history includes uveitis, fatigue, loss of appetite, and weight loss. Active spinal range of motion is limited on flexion and lateral bending. Which of the following is the most likely diagnosis?
Answers:
A. Ankylosing spondylitis
B. Transverse myelitis
C. Multiple sclerosis
D. Metastatic spine disease
E. Guillain-Barre syndrome

A

Ankylosing spondylitis

Discussion:
The history suggests ankylosing spondylitis (AS). AS is a form of arthritis that leads to long-term
inflammation in the joints of the spine and is associated with genetic factors. Most (85%) have
specific human leukocyte antigen HLA-B27. The underlying mechanism is autoimmune or
autoinflammatory, which is associated with other systemic issues including weight loss, fatigue,
uveitis, inflammatory bowel disease, sacroiliitis, and arthropathy of the shoulders and hips. Back
pain is a common complaint and can occur in the setting of a partially fused or entirely fused spine,
either in the neck or the back. Patients with AS can have severe sagittal deformity and require a
major spinal reconstruction in both the cervical spine and thoracic/lumbar spine. AS patients can
also have very severe fractures due to the large moment arms above and below the fracture site.
Cancer is less likely in this patient due to his age and because most pathologic fractures have
mechanical pain, whereas AS pain is not mechanical because the entire spine (or portions of the
spine) are fused. Multiple sclerosis (MS) leads to vision loss and double vision, as well as
relapsing-remitting upper motor neuron symptoms. Guillain-Barre syndrome causes ascending
weakness after a viral infection or vaccination. Transverse myelitis causes acute neurologic deficit
form a spinal cord demyelinating plaque.
References:
Jacobs WB, Fehlings MG. Ankylosing spondylitis and spinal cord injury: origin, incidence,
management, and avoidance. Neurosurgical Focus. 2008; 24(1): E12.
Pubmed Web link: https://pubmed-ncbi-nlm-nih-gov.proxy.library.vanderbilt.edu/18290738/
Teunissen FR, Verbeek BM, Cha TD, Schwab JH. Spinal cord injury after traumatic spine fracture
in patients with ankylosing spinal disorders. J Neurosurg Spine. 2017 Dec;27(6):709-716.
PMID: 28984512
Pubmed Web link: https://pubmed-ncbi-nlm-nih-gov.proxy.library.vanderbilt.edu/28984512/

How well did you know this?
1
Not at all
2
3
4
5
Perfectly
17
Q

A 65-year-old woman undergoes multilevel osteotomies, decompression, T10 to pelvis fixation, and fusion for correction of a kyphoscoliotic deformity. At six months postoperatively, which of the following radiographic parameters best correlates with improved clinical outcome?
Answers:
A. Lumbosacral fractional curve
B. Sacral slope
C. Sagittal vertical axis
D. Pelvic incidence
E. Thoracolumbar/lumbar Cobb angle

A

Sagittal vertical axis

Discussion:
The answer to this question is the sagittal vertical axis (SVA). The patient has kyphoscoliosis –
kypho (kyphotic area of her spine) and scoliosis (a curvature to the bottom of the
thoracolumbar/lumbar spine). Since she was treated with a T10-pelvis fusion, her scoliosis likely
does not extend rostral to the lumbar spine. The fact that she had kyphosis likely means she is
leaning forward, which puts stress on the lumbar musculature and is associated with low back
pain. Several papers have documented that sagittal alignment (how far your head or C7 is over
your femoral heads and pelvis), is associated with quality-of-life metrics. There are other pelvic
parameters that are also associated with quality-of-life metrics and clinical outcome, such as pelvic
tilt, but this is not an answer choice here. If the pelvic tilt is high (>20°-25°), patients are likely
retroverting their pelvis, which indicates they are compensating for sagittal malalignment. The T1-
pelvic angle (T1PA) is another surrogate for sagittal alignment. For this question, sagittal
alignment, measured by SVA, is the answer here. Other measurements certainly affect quality of
life, but less so than the SVA. For example, global coronal alignment – coronal vertical axis (CVA)
– is measured by the difference in the coronal C7-plumb line and the central sacral vertical line
(CSVL), a line drawn up from the sacrum. Coronal malalignment can be associated with
decreased quality of life, but research has shown that coronal measurements matter less than
sagittal alignment. The same is true for regional Cobb angle, such as the lumbosacral fractional
curve (coronal Cobb angle from L4-sacrum). This can be associated with foraminal compression
and radiculopathy.
References:
Schwab FJ, Blondel B, Bess S, et al. Radiographical spinopelvic parameters and disability in the
setting of adult spinal deformity: a prospective multicenter analysis. Spine (Phila Pa 1976). 2013
Jun 1;38(13):E803-12.
Pubmed Web link: https://pubmed-ncbi-nlm-nih-gov.proxy.library.vanderbilt.edu/23722572/
Smith JS, Singh M, Klineberg E, Shaffrey CI, Lafage V, Schwab FJ, Protopsaltis T, Ibrahimi D,
Scheer JK, Mundis G Jr, Gupta MC, Hostin R, Deviren V, Kebaish K, Hart R, Burton DC, Bess S,
Ames CP; International Spine Study Group. Surgical treatment of pathological loss of lumbar
lordosis (flatback) in patients with normal sagittal vertical axis achieves similar clinical improvement
as surgical treatment of elevated sagittal vertical axis: clinical article.
J Neurosurg Spine. 2014 Aug;21(2):160-70. PMID: 24766290.

How well did you know this?
1
Not at all
2
3
4
5
Perfectly
18
Q

A 68-year-old woman is evaluated because of progressive neck pain and high cervical myelopathy. Cervical imaging studies are shown in Figures 1-6. Serum alkaline phosphatase and urinary hydroxyproline excretion levels are elevated. In addition to the dorsal decompression with craniocervical internal fixation and fusion shown in Figure 7, which of the following is the most
appropriate adjuvant treatment for this disorder?
Answers:
A. Radiation
B. Colchicine
C. Steroids
D. Methotrexate
E. Zoledronic Acid

A

Zoledronic Acid

Discussion:
The correct answer is Zolendronic Acid. Paget’s disease of the bone is a disorder of accelerated
osteoclastic bone resorption and reactive osteoblastic bone remodeling. The result is overall
weaker bone that can cause pain or lead to fracture with neural element compromise. Diagnosis of
Paget’s disease includes laboratory studies with evidence of elevated serum Alkaline Phosphatase
levels, and elevated urinary hydroxyproline. A bone scan can reveal increased uptake in affected
areas, with monostotic and polyostotic subtypes. In the spine, the lumbar region (L4 and L5) is the
most frequently involved region (58%), more so than the thoracic (45%) and the cervical spine.
The vertebral body is almost always affected with varying involvement of the posterior bony
elements. CT shows evidence of bone expansion and remodeling, while MRI shoes replacement
of bone marrow. On x-rays and CT, affected vertebrae are sclerotic and dense, a so called “ivory
vertebra” appearance. Osteosarcoma transformation is very rare in the spine (0.7%).
Bisphosphonates are the mainstay of treatment, with the second line being Calcitonin in patients
with bisphosphonate intolerance. Surgical intervention is indicated for neural element compression
or fracture associated spinal instability. Pre and post operative bisphosphonate therapy is
recommended. There is a 25% risk of perioperative complications in this population.
Radiation, steroids, Colchicine, and Methotrexate have no role in Paget’s disease.
References:
Reid IR, Lyles K, Su G, Brown JP et al. A single infusion of zoledronic acid produces sustained
remissions in Paget disease: data to 6.5 years. J Bone Miner Res. 2011 Sep;26(9):2261-70. doi:
10.1002/jbmr.438.
Pubmed Web link: https://pubmed.ncbi.nlm.nih.gov/21638319/
Jorge-Mora A, Amhaz-Escanlar S, Lois-Iglesias A, et al. Surgical treatment in spine Paget’s
disease: a systematic review. Eur J Orthop Surg Traumatol . 2016 Jan;26(1):27-30.
Pubmed Web link: https://pubmed.ncbi.nlm.nih.gov/26126588/
Dell’Atti C, Cassar-Pullicino VN, Lalam RK et al. The spine in Paget’s disease. Skeletal Radiol.
2007 Jul;36(7):609-26.
Pubmed Web link: https://pubmed.ncbi.nlm.nih.gov/17410356/

How well did you know this?
1
Not at all
2
3
4
5
Perfectly
19
Q

A 16-year-old girl is evaluated because of pain in the lower back and right leg. She reports urinary frequency and urgency. Photographs of her lower back are shown. Which of the following spinal anomalies is the most likely cause of her symptoms?
Answers:
A. Teratoma
B. Spinal dermal sinus
C. Normal variant
D. Tethered cord
E. Split cord malformation

A

Split cord malformation

Discussion:
Two-thirds of patients with split cord malformation have overlying skin abnormalities such as tuft of
hair (hypertrichosis), nevi, lipomas, dimples, and hemangiomas. All these cutaneous abnormalities
should trigger work-up of this pathology clinically. Sacral appendages are associated with spinal
teratomas, while coccygeal pits are normal variants (though patients with this abnormality should
be evaluated for any underlying lesions). Y-shaped clefts and sacral dimples can signify tethered
cords or dermal sinus tracts, respectively.
There are two types of split cord malformation. In Type I SCM, the spinal cord is formed of
hemicords; each has its own central canal, pia, and dura. In type I SCM, both dural tubes are
separated by a rigid osseocartilaginous (bony) median septum. Type II SCM (which is referred to
as diplomyelia) is also formed of two hemicords within a single dura, separated by a nonrigid
fibrous median septum. Both groups are associated with spina bifida occulta, and in both groups
each hemicord has its own nerve roots. Treatment includes untethering of the cord at the level of
the spina bifida occulta and occasionally at the level of the split. In type I SCM, never untether the
cord until the bony septum is removed to avoid retraction of the cord against the bony septum.
References:
Mahapatra AK, Gupta DK. Split cord malformations: a clinical study of 254 patients and a proposal
for a new clinical-imaging classification. J Neurosurg. 2005 Dec;103(6 Suppl):531-6. doi:
10.3171/ped.2005.103.6.0531. PMID: 16383252.
Web link: https://pubmed.ncbi.nlm.nih.gov/16383252/
Pang D, Dias MS, Ahab-Barmada M. Split cord malformation: Part I: A unified theory of
embryogenesis for double spinal cord malformations. Neurosurgery. 1992 Sep;31(3):451-80. doi:
10.1227/00006123-199209000-00010. PMID: 1407428.
Pubmed Web link: https://pubmed.ncbi.nlm.nih.gov/1407428/

How well did you know this?
1
Not at all
2
3
4
5
Perfectly
20
Q

Which of the following is the most common cause of an incomplete cervical central cord syndrome?
Answers:
A. Penetrating trauma
B. Disc herniation
C. Tumor
D. Distraction injury
E. Hyperextension injury

A

Hyperextension injury

Discussion:
Central cord syndrome is usually a result of acute hyperextension injury in older patients in the
presence of osteophytic spurs and redundant ligamentum flavum. The injury most often occurs
with a blow to the upper face or forehead. The syndrome is thought to result from edema within the
vascular watershed zone in the central region of the spinal cord. Within the corticospinal tracts, the
upper extremities are represented more medial than the lower extremities. Consequently, patients
classically present with significantly greater motor deficit in the upper extremities than the lower.
Sensory disturbance is variable and can be present below the level of the lesion due to the
ascending spinothalamic tracts.
Anterior cord syndrome is often secondary to a dislocated bone fragment or traumatic disk
herniation resulting in anterior cord compression or occlusion of the anterior spinal artery. This
presents with loss of motor, pain, and temperature below the level of the lesion with preservation of
dorsal column functions. Posterior cord syndrome, which is relatively rare, is due to selective injury
or ischemia to the dorsal columns, and produces sensory ataxia, as well as decreased vibration
and light touch. Mild weakness of the upper extremities can also occur. Brown-Sequard syndrome
is also referred to as spinal cord hemisection and is usually secondary to penetrating rather than
blunt trauma. The findings include ipsilateral motor paralysis, loss of dorsal column function below
the level of the lesion, and contralateral loss of pain and temperature sensation 1-2 segments
below the lesion.
References:
Avila MJ, Hurlbert RJ. Central Cord Syndrome Redefined. Neurosurg Clin N Am. 2021
Jul;32(3):353-363. doi: 10.1016/j.nec.2021.03.007. Epub 2021 May 7. PMID: 34053723.
Pubmed Web link: https://pubmed.ncbi.nlm.nih.gov/34053723/
Hashmi SZ, Marra A, Jenis LG, Patel AA. Current Concepts: Central Cord Syndrome. Clin Spine
Surg. 2018 Dec;31(10):407-412. doi: 10.1097/BSD.0000000000000731. PMID: 30346310.
Pubmed Web link: https://pubmed.ncbi.nlm.nih.gov/30346310/

How well did you know this?
1
Not at all
2
3
4
5
Perfectly
21
Q

During orbitozygomatic osteotomy, which of the following nerve branches is most likely to require release from its bony foramen?
Answers:
A. Supraorbital nerve
B. Abducens nerve
C. Abducens nerve
D. Oculomotor nerve
E. Frontalis branch of the facial nerve

A

Supraorbital nerve

Discussion:
During orbitozygomatic craniotomy, the limits of exposure include the supraorbital notch medially
and the frontozygomatic suture laterally. If a more medial orbitotomy is required, the supraorbital
nerve can be mobilized from its foraminal notch and retracted with the scalp. The frontalis branch
of the facial nerve is protected during elevation of the scalp flap, most effectively with a subfascial
technique near the subgaleal fat pad. The supraorbital nerve provides sensory innervation for the
frontal scalp, forehead, and upper eyelid, and injury to the nerve can result in supraorbital
neuralgia, with intense, unilateral pain the forehead and superior orbit, often with a fluctuating
course.
References:
Modifications to the orbitozygomatic approach. Technical note. J Neurosurg. 2003.
Nov;99(5):924-930.
Pubmed Web link: https://pubmed.ncbi.nlm.nih.gov/14609176/
The work horse of skull base surgery: orbitozygomatic approach. Technique, modifications, and
application. Neurosurg Focus. 2008;25(6):E4.
Pubmed Web link: https://pubmed.ncbi.nlm.nih.gov/19035702/
Sjaastad O, Petersen HC, and Bakketeig LS. Supraorbital neuralgia. Vaga study of headache
epidemiology. Cephalgia, 2005. 25(4):296-304.
Pubmed Web link: https://pubmed.ncbi.nlm.nih.gov/15773827/

How well did you know this?
1
Not at all
2
3
4
5
Perfectly
22
Q

Which of the following structures forms the anterosuperior margin of the foramen of Monro?
Answers:
A. Fornix
B. Septum pellucidum
C. Choroid plexus
D. Thalamus
E. Roof of the third ventricle

A

Fornix

Discussion:
The foramen of Monro is located at the junction of the third ventricle roof and anterior wall. It is
bounded anterosuperiorly by the junction of the column and body of the fornix, and by the
thalamus posteriorly. The choroid plexus, distal branches of the medial posterior choroidal arteries,
thalamostriate vessels, superior choroidal arteries and septal veins all pass through the foramen of
Monro. Injury to the fornix can occur during a third ventriculostomy, though there are limited reports
in the literature. In a series of 94 patients undergoing third ventriculostomy, gross injury to the
fornix was observed in 5 patients; while injury to the right fornix can result in impaired visuo-spatial
memory, these 5 patients had no clinical symptoms. These injuries primarily occurred when placing
the rigid endoscope through a sheath that was placed blindly and too deep, allowing for injury
during passage of the endoscope. Additionally, care should be taken intraoperatively for stability of
the endoscope so as to avoid any traction on the fornix.
References:
The foramen of Monro: a review of its anatomy, history, pathology, and surgery. Childs Nerv Syst.
2014. Oct;30(1):1645-9.
Pubmed Web link: https://pubmed.ncbi.nlm.nih.gov/25079886/
Microsurgery of the third ventricle: Part I. Microsurgical anatomy. Neurosurgery. 1981.
Mar;8(3):334-56.
Pubmed Web link: https://pubmed.ncbi.nlm.nih.gov/7242883/
Kehler U, Regelsberger J, and Gliemroth J. The mechanism of fornix lesions in 3rd
ventriculostomy. Minim Invasive Neurosurg, 2003. 46(4):202-4.
Pubmed Web link: https://pubmed.ncbi.nlm.nih.gov/14506562/

How well did you know this?
1
Not at all
2
3
4
5
Perfectly
23
Q

A 3-month-old girl is being evaluated because of a midline lumbar dimple. An MR scan of the spine is shown. She is at greatest risk for which of the following conditions?
Answers:
A. Hydrocephalus
B. Syringomyelia
C. Scoliosis
D. Meningitis
E. Tethered cord

A

Meningitis

Discussion:
T2-weighted sagittal MRI imaging of the whole spine shows a sacral dermal sinus (hypointense
tract) coursing cephalad as it dives inward from the skin surface. Lumbosacral dermal sinuses are
a potential pathway for intradural infection with a significant risk of bacterial meningitis (76.4%). It
may even result in an intrathecal abscess. Furthermore, the sinus lining contains normal skin
appendages that may result in hair, desquamated epithelium, sebum, and cholesterol within the
tract. If the skin appendages enter the dura, it can irritate the dura leading to sterile chemical
meningitis. Although 25% of sacral dermal sinuses regress to just a dimple, it is recommended to
explore and excise the lumbosacral dermal sinuses before developing signs of infection or
neurologic deficit. The surgical technique consists of an ellipse cut around the sinus opening then
dissection around the sinus tract until its termination. If the tract penetrates into the spinal canal,
laminectomy is performed. If the tract continues intradurally, the dura is opened and inspected for
any intradural lesions. If a dermoid cyst is present, debulking from within is the method of choice to
avoid spilling cyst contents intradurally to avoid recurrence, chemical meningitis, and arachnoiditis.
References:
Au H. Recurrent meningitis in a child due to an occult spinal lesion. CMAJ. 2006;175(7):737.
doi:10.1503/cmaj.060163
Web link: https://www.ncbi.nlm.nih.gov/pmc/articles/PMC1569937/
Powell KR, Cherry JD, Hougen TJ, Blinderman EE, Dunn MC. A prospective search for congenital
dermal abnormalities of the craniospinal axis. J Pediatr. 1975 Nov;87(5):744-50. doi:
10.1016/s0022-3476(75)80298-8. PMID: 1185339.
Pubmed Web link: https://pubmed.ncbi.nlm.nih.gov/1185339/

How well did you know this?
1
Not at all
2
3
4
5
Perfectly
24
Q

Which of the following is the most effective treatment for an isolated type II odontoid fracture in an otherwise healthy 60-year-old man?
Answers:
A. Treatment only if the patient has neck pain
B. Surgical fixation
C. Nonoperative management in a rigid cervical collar
D. Nonoperative management in halo immobilization
E. Nonoperative management in a soft cervical collar with serial radiographic evaluation

A

Surgical fixation

Discussion:
There is no Class I medical evidence on the management of patients with acute traumatic odontoid
fractures.
Class II medical evidence: risk of nonunion of a type II odontoid fracture in patients ≥ 50 years of
age is 21 times greater than for younger patients with a similar type II odontoid fracture. Therefore,
consideration of surgical stabilization and fusion for type II odontoid fractures in patients ≥ 50 years
of age is recommended.
Treatment of type II odontoid fractures with a cervical collar alone or traction followed by cervical
collar immobilization is associated with lower fracture union rates.
If nonoperative treatment is undertaken, factors associated with type II odontoid fracture nonunion
are any of the following: age greater than 50, dens displacement of ≥ 5 mm, change in angulation
of the odontoid fracture by ≥ 5° on lateral radiography taken at 2 weeks after immobilization,
comminuted fracture pattern, or inability to maintain fracture alignment with external orthosis.
The management of odontoid fractures in elderly patients is associated with increased failure
rates, and higher rates of morbidity and mortality compared to younger patients irrespective of the
treatment offered.
References:
Ryken TC, Hadley MN, Aarabi B, Dhall SS, Gelb DE, Hurlbert RJ, Rozzelle CJ, Theodore N,
Walters BC. Management of isolated fractures of the axis in adults. Neurosurgery. 2013 Mar;72
Suppl 2:132-50. doi: 10.1227/NEU.0b013e318276ee40. PMID: 23417186.
Pubmed Web link: https://pubmed.ncbi.nlm.nih.gov/23417186/
Rizvi SAM, Helseth E, Rønning P, Mirzamohammadi J, Harr ME, Brommeland T, Aarhus M,
Høstmælingen CT, Ølstørn H, Rydning PNF, Mejlænder-Evjensvold M, Utheim NC, Linnerud H.
Odontoid fractures: impact of age and comorbidities on surgical decision making. BMC Surg. 2020
Oct 14;20(1):236. doi: 10.1186/s12893-020-00893-7. PMID: 33054819; PMCID: PMC7556921.
Pubmed Web link: https://pubmed.ncbi.nlm.nih.gov/33054819/

How well did you know this?
1
Not at all
2
3
4
5
Perfectly
25
Q

Which of the following factors most commonly determines instability of C1 fracture (Jefferson fracture)?
Answers:
A. Number of fracture sites in the C1 ring
B. Transverse ligament integrity
C. Concomitant C2 pars fracture
D. Concomitant odontoid fracture
E. Concomitant occipital condylar fracture

A

Transverse ligament integrity

Discussion:
A Jefferson fracture is a burst-type fracture of the C1 ring caused by an axial loading injury. The
mechanical stability of a Jefferson fracture depends largely on the integrity of the transverse
atlantal ligament (TAL). Two common mechanisms to assess the integrity of the transverse
ligament are the rule of Spence and Dickman’s criteria.
The rule of Spence is a method for assessing the transverse ligament by measuring the sum of the
overhang of the lateral masses of C1 over C2 on AP/open-mouth odontoid view x-ray or on
coronal CT scan. If the sum overhang (left and right added together) is greater than or equal to
7mm then the transverse ligament is presumed to be disrupted.
Dickman’s criteria utilizes fine-cut CT scan and MRI to evaluate injury to the transverse ligament.
Type 1 injuries have visual damage to the ligament fibers and are recommended to be treated
surgically due to the decreased likelihood of ligamentous healing. Type II injuries have avulsion of
the ligament at the C1 osseous insertion site, and over 70% of these healed with external cervical
orthosis treatment.
A hangman’s fracture is a bilateral fracture through the pars of C2 and is characterized by the
extent of C2-3 displacement and/or angulation, as well as the presence of C2-3 facet or disc
disruption. Odontoid fractures involve a fracture of a portion of the dens and are classified by a
fracture line through either the dens tip (type I), base (type II), or the C2 body (type III). Occipital
condyle fractures are classified as non-displaced linear condyle fracture (type I), extension of a
basilar skull fracture (type II), or condyle fracture with alar ligament avulsion (type III). None of
these other C1 or C2 fractures play a role in determining the stability of a C1 fracture.
References:
Liu P, Zhu J, Wang Z, Jin Y, Wang Y, Fan W, Liu M, Zhao J. “Rule of Spence” and Dickman’s
Classification of Transverse Atlantal Ligament Injury Revisited: Discrepancy of Prediction on
Atlantoaxial Stability Based on Clinical Outcome of Nonoperative Treatment for Atlas Fractures.
Spine (Phila Pa 1976). 2019 Mar 1;44(5):E306-E314. doi: 10.1097/BRS.0000000000002877.
PMID: 30222691.
Pubmed Web link: https://pubmed.ncbi.nlm.nih.gov/30222691/
Woods RO, Inceoglu S, Akpolat YT, Cheng WK, Jabo B, Danisa O. C1 Lateral Mass Displacement
and Transverse Atlantal Ligament Failure in Jefferson’s Fracture: A Biomechanical Study of the
“Rule of Spence”. Neurosurgery. 2018 Feb 1;82(2):226-231. doi: 10.1093/neuros/nyx194. PMID:
28431136.
Pubmed Web link: https://pubmed.ncbi.nlm.nih.gov/28431136

How well did you know this?
1
Not at all
2
3
4
5
Perfectly
26
Q

A 24-year-old woman is evaluated for right arm and leg weakness and left abducens nerve palsy immediately after being involved in a motor vehicle collision. X-ray film of the lateral cervical spine shows pre-vertebral soft tissue swelling of the upper cervical spine and a basion-dental interval of 16 mm. CT scan of the head shows no intracranial hemorrhage. Which of the following is the most appropriate management of this patient?
Answers:
A. Flexion-extension xrays to assess for instability
B. Halo placement
C. Rigid cervical collar placement
D. Operative intervention for internal fixation and arthrodesis
E. Obtain MRI brain

A

Operative intervention for internal fixation and arthrodesis

Discussion:
This patient is exhibiting clinical and radiographic signs of atlanto-occipital dislocation (AOD). The
most appropriate treatment option is internal fixation and arthrodesis. Traumatic AOD frequently
results in death at the time of injury. Surviving patients may present with a normal neurological
exam, but often present with hemiparesis or quadriparesis.
Cranial nerve palsies may also be seen due to traction on the brainstem. This injury is extremely
unstable and patients frequently deteriorate without timely occipitocervical instrumentation.
Traynelis has classified these injuries as Type I (anterior), Type II (longitudinal), Type III (posterior),
and other types (lateral, rotational, or multidirectional). The basion-axial-interval (BAI)-basiondental-interval (BDI) of Harris is an accepted method of diagnosing atlanto-occipital dislocation on
a lateral cervical x-ray. The BAI is the distance from the basion to a line drawn along the posterior
wall of the C2 vertebral body. A distance of more than +12 mm indicates a Type I injury while a
distance of -4 mm or more indicates a Type III injury. A BDI greater than 12mm is also abnormal
and is consistent with a Type II AOD. An additional finding on imaging studies is upper cervical
prevertebral soft tissue swelling.
Because of the degree of dislocation and instability associated with this injury, external bracing
(including halo) is a poor management option. The use of traction in these highly unstable injuries
is generally contraindicated and can lead to death. The best management option is immediate
operative intervention for occipito-cervical instrumentation and arthrodesis.
References:
Theodore N, Aarabi B, Dhall SS, Gelb DE, Hurlbert RJ, Rozzelle CJ, Ryken TC, Walters BC,
Hadley MN. The diagnosis and management of traumatic atlanto-occipital dislocation injuries.
Neurosurgery. 2013 Mar;72 Suppl 2:114-26. doi: 10.1227/NEU.0b013e31827765e0. PMID:
23417184.
Pubmed Web link: https://pubmed.ncbi.nlm.nih.gov/23417184/
Joaquim AF, Schroeder GD, Vaccaro AR. Traumatic Atlanto-Occipital Dislocation-A
Comprehensive Analysis of All Case Series Found in the Spinal Trauma Literature. Int J Spine
Surg. 2021 Aug;15(4):724-739. doi: 10.14444/8095. Epub 2021 Jul 21. PMID: 34289992; PMCID:
PMC8375687.
Pubmed Web link: https://pubmed.ncbi.nlm.nih.gov/34289992/

How well did you know this?
1
Not at all
2
3
4
5
Perfectly
27
Q

A 61-year-old man is evaluated for an L5-S1 far lateral disc herniation. Which of the following clinical findings is most likely in this patient?
Answers:
A. Weakness in the extensor hallucis longus
B. Diminished patellar responses
C. Pain in the anterior thigh
D. Weakness in the quadriceps
E. Weakness in the gastrocnemius

A

Weakness in the extensor hallucis longus

Discussion:
Of the answer choices presented, weakness in the extensor hallucis longus is the most likely
clinical finding in a patient with a far lateral L5-S1 disc herniation. A far lateral disc will affect the
exiting nerve root at the level described, i.e. L5 in this case. A more central pathology within the
canal will likely affect the traversing nerve root. The traversing nerve root in this patient would have
been S1, and may result in weakness in the gastrocnemius. An affected patellar response,
weakness in the quadriceps muscles, and pain in the anterior thigh all describe symptoms of the
L4 nerve root being affected.
References:
Voyadzis JM, Gala VC, Sandhu FA, Fessler RG. Minimally invasive approach for far lateral disc
herniations: results from 20 patients. Minim Invasive Neurosurg. 2010 Jun;53(3):122-126. Rhee
JM, Schaufele M, Abdu WA. Radiculopathy and the herniated lumbar disc. Controversies
regarding pathophysiology and management. J Bone Joint Surg Am. 2006 Sep;88(9):2070-2080.
Pubmed Web link: https://www.thieme-connect.com/products/ejournals/abstract/10.1055
/s-0030-1249102
Rhee JM, Schaufele M, Abdu WA. Radiculopathy and the herniated lumbar disc. Controversies
regarding pathophysiology and management. J Bone Joint Surg Am. 2006 Sep;88(9):2070-2080.
Pubmed Web link: https://journals-lww-com.geihsl.idm.oclc.org/jbjsjournal/Fulltext/2006/09000
/Radiculopathy_and_the_Herniated_Lumbar_Disc_.23.aspx
Epstein, N. Foraminal and far lateral lumbar disc herniations: surgical alternatives and outcome
measures. Spinal Cord 40, 491–500 (2002). https://doi.org/10.1038/sj.sc.3101319

28
Q

An 86-year-old woman with a history of chronic obstructive pulmonary disease, aortic stenosis, and severe dementia resides in a nursing home. After falling, she has neck pain, but no neurological deficits. A sagittal reconstruction CT scan of the cervical spine is shown. Which of the
following is the most appropriate treatment?
Answers:
A. Halo immobilization
B. Occiput – C2 instrumented fusion
C. C1-2 posterior instrumented fusion
D. Odontoid screw
E. Hard collar immobilization with serial follow-up xrays

A

Hard collar immobilization with serial follow-up xrays

Discussion:
The risk-benefit profile for surgical intervention of type II odontoid fractures in the elderly
population is an important consideration. While surgical intervention may yield immediate
stabilization of the fracture, one has to consider the potential collateral damage of surgical blood
loss, anesthesia, and post-operative opioids. The patient in this particular case is 86 years old, and
has a history of dementia, COPD, and cardiac pathology. A much more practical approach to this
case would be to strongly consider hard collar immobilization with follow-up serial xrays. Chronic
neck immobilization also harbors risks as well, such as the contribution it may lend to temporary
swallowing dysfunction. A lack of dynamic motion on xrays and cessation of mechanical neck pain
may direct the surgeon to remove the external orthosis. Although halo immobilization is an option,
this has potential to cause significant quality of life concerns in this age population, and may
actually lend to a worse overall outcome.
References:
Koech F, Ackland HM, Varma DK, Williamson OD, Malham GM. Nonoperative management of type
II odontoid fractures in the elderly. Spine (Phila Pa 1976). 2008 Dec 15;33(26):2881-6. doi:
10.1097/BRS.0b013e31818d5407. PMID: 19092619
Pubmed Web link: https://pubmed.ncbi.nlm.nih.gov/19092619/
Azad TD, Jiang B, Zhu AM, Theodore N. Healing of type II odontoid fracture without surgery in an
octogenarian - Case report and literature review. J Clin Neurosci. 2019 Jun;64:23-24. doi:
10.1016/j.jocn.2019.03.049. Epub 2019 Mar 30. PMID: 30940454
Pubmed Web link: https://pubmed.ncbi.nlm.nih.gov/30940454/

29
Q

A 46-year-old man comes to the emergency department with progressive neck pain and transient quadriplegia after falling in his kitchen. Currently, he is moving all extremities and muscle strength is 4/5. He has a history of hypertension and has never had surgery but had two teeth extracted
about two months ago. A CT scan of the cervical spine is shown. Which of the following is the most likely diagnosis?
Answers:
A. Osteomyelitis
B. Klippel-Feil syndrome
C. Metastatic spinal tumor
D. Primary spinal tumor
E. Rheumatoid arthritis

A

Osteomyelitis

Discussion:
Given the patient’s history of a recent tooth abstraction, osteomyelitis is the most likely diagnosis.
Though patients with rheumatoid arthritis can have C2 erosion, this is often at the C1-2 articular
surfaces and is associated with pannus formation. The teeth are in close approximation to the
pharynx and upper cervical spine, which can lead to direct extension of oral or pharyngeal
infections. In this scenario, the patient likely had traumatic injury to an already compromised C2
vertebrae, with resulting fracture and odontoid translation, leading to acute spinal cord injury. A
spinal metastasis can also lead to bony erosion of C2 and epidural disease-causing cord
compression, but given the recent tooth extraction history, infection is the most likely cause. Other
risk factors for infection like this would be immune suppression, poorly controlled diabetes mellitus,
chronic oral steroid use, intravenous drug use, or smoking. Klippel-Feil is a congenital abnormality
with autofusion of multiple cervical vertebral levels, and not what this patient has.
References:
Noguchi S, Yanaka K, Yamada Y, Nose T. Diagnostic pitfalls in osteomyelitis of the odontoid
process: case report. Surg Neurol. 2000 Jun;53(6):573-578; discussion 578-579.
Pubmed Web link: https://pubmed-ncbi-nlm-nih-gov.proxy.library.vanderbilt.edu/10940425/
Barnes B, Alexander JT, Branch CL Jr. Cervical osteomyelitis: a brief review. Neuros

30
Q

A 76-year-old man who fell from ground height is evaluated in the emergency department for head lacerations and mild neck pain. Physical examination shows mild tenderness of the posterior neck region with limitation of cervical motion. Neurological examination shows no abnormalities. X-ray films of the cervical spine are shown in Figures 1 and 2. Which of the following is the most appropriate next step in management of this patient?
Answers:
A. Discharge from the ED with follow up in 2 weeks
B. MRI Cervical Spine
C. CT Cervical Spine
D. Gardner Wells traction
E. Cervical Collar Immobilization

A

Cervical Collar Immobilization

Discussion:
Diffuse Idiopathic Skeletal Hyperostosis (DISH), also known as Forestier’s disease, is a noninflammatory progressive calcification of ligaments and tendons of the spine due to an unknown
aetiology. DISH would typically present in an older male (60-70 yr old) and would involve both the
anterior (ALL) and posterior longitudinal ligaments (PLL) but would spare the SI joints and facet
joints. DISH may present with dysphagia or hoarseness due to anterior cervical osteophytes, or
chronic neck or back stiffness and pains.
Ankylosing Spondylitis (AS), on the other hand, is an inflammatory disease, most common in
young (30 yr) males, with HLA-B27 association. The classic appearance of Bamboo spine is due
to involvement of both ALL and PLL in addition to the disc spaces. SI and facet joints are involved,
with possible extra-spinal (eg. ocular, GI, cardiac) manifestations.
Given the rigidity of the spine in patients with DISH, a high level of suspicion is warranted even in
low energy trauma. An X ray C Spine is not enough to rule out a fracture, and a CT scan would be
an adequate study. MRI may be utilized in patients with non-ankylosed levels in which a discoligamentous injury may have occurred, and in patients with neurological deficits.
The first step though is cervical collar immobilization until imaging is obtained. There is no role for
Gardner-Wells traction in this case.
References:
Whang PG, Goldberg G, Lawrence JP, et al. The management of spinal injuries in patients with
ankylosing spondylitis or diffuse idiopathic skeletal hyperostosis: a comparison of treatment
methods and clinical outcomes. J Spinal Disord Tech. 2009 Apr;22(2):77-85.
Pubmed Web link: https://pubmed.ncbi.nlm.nih.gov/19342927/
Rustagi T, Drazin D, Oner C et al. Fractures in Spinal Ankylosing Disorders: A Narrative Review of
Disease and Injury Types, Treatment Techniques, and Outcomes. J Orthop Trauma. 2017 Sep;31
Suppl 4:S57-S74.
Pubmed Web link: https://pubmed.ncbi.nlm.nih.gov/28816877/
Shah NG, Keraliya A, Harris MB et al. Spinal trauma in DISH and AS: is MRI essential following
the detection of vertebral fractures on CT? Spine J. 2021 Apr;21(4):618-626.
Pubmed Web link: https://pubmed.ncbi.nlm.nih.gov/33130303/

31
Q

A 45-year-old woman has recurrent back and leg pain six weeks after undergoing L5-S1 transforaminal lumbar interbody fusion, which included rhBMP2 (recombinant human bone morphogenetic protein 2). She is afebrile. White blood cell count is 11,000/mm3, erythrocyte sedimentation rate is 24 mm/h, and serum C-reactive protein level is 16 mg/L. MR images are shown. Which of the following is the most likely diagnosis?
Answers:
A. Abscess
B. Seroma
C. Tumor
D. Hematoma
E. Pseudomeningocele

A

Seroma

Discussion:
A common complication after the use of BMP is a postoperative seroma. BMP use has been
associated with seroma formation post-operatively due to an inflammatory response by the body.
This inflammatory response may cause a rise in erthyrocyte sedimentation rate (ESR), C-reactive
protein (CRP), and the white blood cell count (WBC). These seromas often do not commonly
cause symptomatic compression of the spinal cord, cauda equina, or nerve roots, but they
certainly can be radiologically compressive. It is a hotly debated topic whether use of BMP leads to
an increased risk of complications, including seroma formation. The only on-label approval for
BMP in the spine is during anterior lumbar interbody fusion (ALIF) procedures, but it is commonly
used off-label during other lumbar fusion surgeries. There are numerous reported complications in
the literature associated with BMP use in the spine. Other important complications associated with
BMP – in addition to seroma – are the following: 1) resorption and/or subsidence after an interbody
procedure, also referred to as osteolysis; 2) ectopic bone formation causing neurologic deficit; 3)
graft malpositioning or loosening; 4) infection; 5) retrograde ejaculation; and 6) carcinogenicity
(though causing cancer has been strongly refuted in recent years).
References:
Epstein NE. Pros, cons, and costs of INFUSE in spinal surgery. Surg Neurol Int. 2011 Jan 24;2:10.
Pubmed Web link: https://pubmed-ncbi-nlm-nih-gov.proxy.library.vanderbilt.edu/21297932/
Faundez A, Tournier C, Garcia M, Aunoble S, Le Huec C. Bone morphogenic protein use in spine
surgery-complications and outcomes: a systematic review. International Orthopaedics. 2016.
Jun;40(6):1309-19.
Pubmed Web link: https://pubmed-ncbi-nlm-nih-gov.proxy.library.vanderbilt.edu/26961193/

32
Q

Cells from which of the following areas give rise to the ascending lateral spinothalamic tract?
Answers:
A. Anterior Horn Cells
B. Substantia Gelatinosa
C. Nucleus Proprius
D. Reticular Formation
E. Dorsal Root Ganglion

A

Nucleus Proprius

Discussion:
The ascending spinothalamic tract relays pain and temperature sensations. While first order
pseudo-unipolar neurons are in the dorsal root ganglion (DRG), their axons pass through Rexed
lamina II of the dorsal horn (Substantia Gelatinosa), which is rich in interneurons, before ultimately
synapsing with the 2nd order neurons in Rexed lamina III –V. Nucleus Proprius corresponds to
Rexed lamina III-IV. It is these neuron fibers that cross to the opposite side of the cord and ascend
forming the lateral spinothalamic tract, and project to the thalamic (3rd order neuron) nuclei which
subsequently project to the sensorimotor cortex.
Anterior horn cells give rise to second order neurons in the cortico-spinal tract. The reticular
formation contributes to the indirect pathway for pain and temperature via synapses of crossed
second order neurons in the dorsal horns. These neurons ultimately project to the thalamus via the
reticulothalamic tract.
References:
Carpenter MB. Core Text of Neuroanatomy. 4th ed. Baltimore: Williams and Wilkins; 1991: 86-87.
Pain and Disability: Clinical, Behavioral, and Public Policy Perspectives. Institute of Medicine (US)
Committee on Pain, Disability, and Chronic Illness Behavior; Osterweis M, Kleinman A, Mechanic
D, editors. Washington (DC): National Academies Press (US); 1987.
Pubmed Web link: https://www.ncbi.nlm.nih.gov/books/NBK219252/

33
Q

The findings in the sagittal (Figure 1) and axial (Figure 2) MR images shown are most commonly associated with which of the following cutaneous abnormalities?
Answers:
A. Y shaped cleft
B. Coccygeal Pit
C. Sacral Dimple
D. Sacral Appendage
E. Tuft of Hair

A

Tuft of Hair

Discussion:
Two-thirds of patients with diastematomyelia (split cord malformation type I) have overlying skin
abnormalities such as tuft of hair (hypertrichosis), nevi, lipomas, dimples, and hemangiomas. All
these cutaneous abnormalities should trigger work-up of this pathology clinically. There are other
spine abnormalities at the level of the spinal cord splitting, such as absent disc and dorsal
hypertrophic bone (spike) where the median ridge attaches. Furthermore, these patients often
have high arched feet. Sacral appendages are associated with spinal teratomas, while coccygeal
pits are normal variants (though patients with this abnormality should be evaluated for any
underlying lesions). Y-shaped clefts and sacral dimples can signify tethered cords or dermal sinus
tracts, respectively.
References:
Pang D. Split cord malformations. In: Pang D, editor. Disorders of the Pediatric Spine. New York:
Raven Press; 1995. pp. 203–52.
Pang D, Dias MS, Ahab-Barmada M. Split cord malformation: Part I: A unified theory of
embryogenesis for double spinal cord malformations. Neurosurgery. 1992 Sep;31(3):451-80. doi:
10.1227/00006123-199209000-00010. PMID: 1407428.
Pubmed Web link: https://pubmed.ncbi.nlm.nih.gov/1407428/

34
Q

In a 72-year-old man with a type II odontoid fracture, which of the following factors best predicts the likelihood of developing a nonunion with nonoperative management?
Answers:
A. Age > 65
B. Associated C1 posterior arch fracture
C. Fracture displacement > 2mm
D. Age > 65, Fracture displacement > 2mm, and Posterior displacement of the fracture > 5mm
E. Posterior displacement of the fracture > 5mm

A

Age > 65, Fracture displacement > 2mm, and Posterior displacement of the fracture > 5mm

Discussion:
Many type II odontoid fractures are treated conservatively with non-surgical management.
However, specific risk factors exist that may predict non-union of the fracture: age > 65, a fracture
gap > 2mm, and posterior displacement of the fracture > 5mm. Associated C1 posterior arch
fracture is not associated with developing non-union. The surgeon should have a discussion with
the patient & family about these risk factors during the decision-making process of operative vs
non-operative treatment.
References:
Harrop JS, Hart R, Anderson PA. Optimal treatment for odontoid fractures in the elderly. Spine
(Phila Pa 1976). 2010 Oct 1;35(21 Suppl):S219-S227.Pryputniewicz DM, Hadley MN. Axis
fractures. Neurosurgery. 2010 Mar;66(3 Suppl):68-82.
Pubmed Web link: https://pubmed.ncbi.nlm.nih.gov/20881465/
Lewis E, Liew S, Dowrick A. Risk factors for non-union in the non-operative management of type II
dens fractures. ANZ J Surg. 2011 Sep;81(9):604-7. doi: 10.1111/j.1445-2197.2010.05586.x. PMID:
22295381
Pubmed Web link: https://pubmed.ncbi.nlm.nih.gov/22295381/

35
Q

A 52-year-old woman is evaluated for a non-displaced traumatic type II odontoid fracture associated with transverse ligament disruption. Which of the following is the most appropriate treatment?
Answers:
A. Occiput to C2 posterior fusion
B. Rigid cervical orthosis and observation with serial xrays
C. Flexion-extension xrays to determine if instability is present
D. C1-C2 posterior fusion
E. Halo placement

A

C1-C2 posterior fusion

Discussion:
Odontoid fractures associated with disruption of the transverse ligament are considered unstable
spinal injuries that necessitate stabilization. The transverse ligament affixes the odontoid process
to the anterior arch of C1 and permits significant cervical rotation to occur (nearly 50% of all
rotation). Disruption of this ligament can result in translation of C1 relative to C2 and possible
neurologic injury. Ligament injuries generally do not heal well, and the odontoid process has a poor
vascular supply when disconnected from the C2 vertebral body. Other features associated with low
likelihood of fracture healing include age (>= 65 years) odontoid fracture displacement width and
odontoid angulation relative to C2. This patient is relatively young (<65), has no noted fracture
displacement, but has injury to the transverse ligament, which makes external cervical orthosis
with a collar or halo unlikely to result in sufficient healing. This injury results in atlanto-axial
instability, not occipitocervical instability, making occiput-to-C2 fusion unindicated. The correct
answer is C1-2 posterior fusion (multiple technique options).
References:
Joaquim AF, Patel AA. Surgical treatment of Type II odontoid fractures: anterior odontoid screw
fixation or posterior cervical instrumented fusion? Neurosurg Focus. 2015 Apr;38(4):E11. doi:
10.3171/2015.1.FOCUS14781. PMID: 25828487.
Pubmed Web link: https://pubmed.ncbi.nlm.nih.gov/25828487/
Shen Y, Miao J, Li C, Fang L, Cao S, Zhang M, Yan J, Kuang Y. A meta-analysis of the fusion rate
from surgical treatment for odontoid factures: anterior odontoid screw versus posterior C1-C2
arthrodesis. Eur Spine J. 2015 Aug;24(8):1649-57. doi: 10.1007/s00586-015-3893-2. Epub 2015
Mar 25. PMID: 25805577.
Pubmed Web link: https://pubmed.ncbi.nlm.nih.gov/25805577

36
Q

A 36-year-old woman presents to the emergency department with an intermittent history of difficulty with urination, anterior tibialis weakness, and diminished rectal tone. An MR image is shown. Which of the following is the most appropriate surgical option?
Answers:
A. Follow-up clinically and radiographically in 3 months
B. Patient reassurance that the condition will improve on loperamide
C. Excision and untethering
D. Refer to a urologist
E. Physiotherapy

A

Excision and untethering

Discussion:
The surgical principles in lipoma of the filum terminale are to preserve neurological tissue, untether
the spinal cord (if present), and debulk the lipomatous mass. It is essential to consider whether the
patient is symptomatic or not as well as the position of the conus when considering surgery. It is
recommended to operate in all symptomatic filum terminale lipoma patients, as the filum terminale
lipoma may be a manifestation of tight terminal filum, a disorder in which a low-lying conus
coexists with a short, hypertrophic filum. It is controversial whether to operate on conus lipoma
patients who are not symptomatic. This patient is symptomatic, so it is recommended to proceed
with lipoma excision and untethering of the cord. All the other options can lead to neurological
function decline while pursuing. Loperamide is an antidiarrheal with weak evidence to improve the
tone in the anal sphincter muscle.
References:
Finn MA, Walker ML. Spinal lipomas: clinical spectrum, embryology, and treatment. Neurosurg
Focus. 2007;23(2):E10. doi: 10.3171/FOC-07/08/E10. PMID: 17961016.
Web link: https://pubmed.ncbi.nlm.nih.gov/17961016/
McLendon RE, Oakes WJ, Heinz ER, Yeates AE, Burger PC. Adipose tissue in the filum terminale:
a computed tomographic finding that may indicate tethering of the spinal cord. Neurosurgery. 1988
May;22(5):873-6. PMID: 2967925.
Pubmed Web link: https://pubmed.ncbi.nlm.nih.gov/2967925/

37
Q

A 55-year-old man is brought to the emergency department after a motor vehicle collision. Plain x-ray films and CT scans of the cervical spine show a type II odontoid fracture posteriorly displaced 5 to 6 mm. The transverse ligament is intact, and the fracture appears reducible. Which of the following treatment modalities offers the highest rate of successful fracture healing?
Answers:
A. Cervical traction followed by hard cervical collar for 12 weeks
B. Cervical traction followed by anterior odontoid screw
C. Cervical traction followed by hard cervical collar for 12 weeks
D. Cervical traction followed by posterior atlanto-axial fixation and fusion
E. Cervical traction followed by halo immobilization for 12 weeks

A

Cervical traction followed by posterior atlanto-axial fixation and fusion

Discussion:
C1-2 fixation and fusion yields the highest rate of successful fracture healing for a displaced Type
II odontoid fracture. Although traction may not be absolutely necessary, it may assist in the
reduction of the displacement prior to instrumentation. Despite the fact that the question stem
states the fracture is reducible, caution needs to be taken when placing an odontoid screw in this
situation. The fractured fragment may be prone to further migration during screw insertion. Rates
of dysphagia after odontoid screw placement become relevant with an aging population. A C2/3
ACDF may be a reasonable strategy to address certain types of Hangman’s fractures. It would not
address the C1-2 instability from a Type II odontoid fracture, though. A hard collar would not yield
the highest rate of fracture healing when compared to other appropriate options. Halo
immobilization after fracture reduction is certainly an option in this case. However, a C1/2 fusion
would be a more definitive method of achieving fracture healing. Halo immobilization also harbors
inherent morbidities with the aging population.
Multiple meta-analyses within the literature have yielded ranges of non-union and fusion rates as it
applies to different treatment modalities for type 2 odontoid fractures. Non-union rates for collar
and halo immobilization have been reported as high as 80% and 63%, respectively. Anterior
odontoid screw fixation yields a fusion rate of 84-96%, while posterior C1/2 screw fixation results in
a nearly 100% fusion rate.
It should be noted that many elderly patients are able to function without significant pain after
achieving a “fibrous” union. In such cases, patients may have a lack of radiographic osseous union
across the fracture line, but maintain radiographic stability on dynamic films.
References:
Ivancic PC. Odontoid fracture biomechanics. Spine (Phila Pa 1976). 2014 Nov
15;39(24):E1403-10. doi: 10.1097/BRS.0000000000000609. PMID: 25271495
Pubmed Web link: https://pubmed.ncbi.nlm.nih.gov/25271495/
Grasso G, Leone L, Torregrossa F. Management of Odontoid Cervical Fracture. World Neurosurg.
2019 Mar;123:246-247. doi: 10.1016/j.wneu.2018.12.038. Epub 2018 Dec 19. PMID: 30579013.
Pubmed Web link: https://pubmed.ncbi.nlm.nih.gov/30579013/

38
Q

A 47-year-old woman presents with bilateral arm paresthesia and neck and shoulder pain since childhood. MR images (T1-weighted (left), STIR (right)) of the cervical spine are shown. Which of the following is the most likely diagnosis?
Answers:
A. Epidermoid cyst
B. Lipoma
C. Tethered cord
D. Teratoma
E. Arachnoid cyst

A

Lipoma

Discussion:
MRI images show a hyperintense lesion on the T1W image that was inverted on the fat
suppression sequence (hypointense). So, the answer is spinal cord lipomas. Teratomas are
commonly heterogeneously hyperintense on T1W and STIR images. Arachnoid cysts will be
hypointense on T1W images and hyperintense on STIR images. Epidermoid cysts are hypointense
on T1W images and hyperintense on STIR images, which are completely opposite to the above
patient. Neurenteric cyst can be shown as hyperintense on T1 due to the proteinaceous material
within the cyst and will be hyperintense on STIR images because it is not of fat composition.
References:
Pruthi N, Devi BI. Nondysraphic cervical and thoracic intraspinal lipomas: a review. Br J
Neurosurg. 2010 Jun;24(3):228-232. doi: 10.3109/02688691003653744. PMID: 20465452.
Web link: https://pubmed.ncbi.nlm.nih.gov/20465452/
Wang H, Huang Y, Nie P, Dong C, Hou F, Hao D, Xu W. MRI findings in intraspinal mature
teratoma. Clin Radiol. 2016 Jul;71(7):717.e1-8. doi: 10.1016/j.crad.2016.04.010. Epub 2016 May
11. PMID: 27180081.
Pubmed Web link: https://pubmed.ncbi.nlm.nih.gov/27180081/
Kwong, Y. Neurenteric cyst. Case study, Radiopaedia.org. (accessed on 01 Feb 2022)
https://doi.org/10.53347/rID-21461
Web link: https://radiopaedia.org/cases/neurenteric-cyst?lang=us
Dembla, S. Pediatric spinal epidermoid cyst. Case study, Radiopaedia.org. (accessed on 01 Feb
2022) https://doi.org/10.53347/rID-94225
Web link: https://radiopaedia.org/cases/pediatric-spinal-epidermoid-cyst?lang=us

39
Q

In newborns with myelomeningocele, which of the following congenital abnormalities is most likely to require surgical intervention during infancy?
Answers:
A. Chiari malformation
B. Syringomyelia
C. Hydrocephalus
D. Tethered spinal cord
E. Scoliosis

A

Hydrocephalus

Discussion:
Patients with myelomeningocele can have multiple other neurological conditions. Of these, the
most likely that would need to be treated from a survivability standpoint would be hydrocephalus to
prevent increased intracranial pressure. 90% of patients with Chiari malformation type II develop
hydrocephalus, and up to 89% of these patients require ventriculoperitoneal shunt placement.
Tethered cord, Chiari malformation, scoliosis, and syringomyelia can also be treated if
symptomatic, though these are less urgent than shunting for hydrocephalus in this patient
population. The prevalence of scoliosis among myelomeningocele patients is around 78%;
however, most symptomatic patients improve with untethering of the cord. Only approximately 37%
of scoliosis in myelomeningocele patients require deformity correction surgery. Posterior fossa
decompression is recommended for Chiari type II malformation presenting with neurogenic
dysphagia, apneic spells, or stridor. It is indicated only in 18.7% of myelomeningocele patients;
however, before recommending decompression, it is recommended to make sure that the shunt is
functioning. The overall prevalence of syringomyelia in myelomeningocele patients is around 48%;
however, most of these patients improve by ventriculoperitoneal shunting and spinal cord
untethering.
References:
Awad AW, Aleck KA, Bhardwaj RD. Concomitant achondroplasia and Chiari II malformation: A
double-hit at the cervicomedullary junction. World J Clin Cases. 2014;2(11):711-716.
doi:10.12998/wjcc.v2.i11.711
Pubmed Web link: https://www.ncbi.nlm.nih.gov/pmc/articles/PMC4233428/
Heyns A, Negrini S, Jansen K, Moens P, Schelfaut S, Peers K, Kiekens C. The prevalence of
scoliosis within Belgian myelomeningocele population and the correlation with ambulatory status
and neurological comorbidities: a chart audit. Spinal Cord. 2021 Oct;59(10):1053-1060. doi:
10.1038/s41393-020-00611-3. Epub 2021 Jan 25. PMID: 33495580.
Pubmed Web link: https://pubmed.ncbi.nlm.nih.gov/33495580/
Park TS, Hoffman HJ, Hendrick EB, Humphreys RP. Experience with surgical decompression of
the Arnold-Chiari malformation in young infants with myelomeningocele. Neurosurgery. 1983
Aug;13(2):147-52. doi: 10.1227/00006123-198308000-00007. PMID: 6888694.
Pubmed Web link: https://pubmed.ncbi.nlm.nih.gov/6888694/
La Marca F, Herman M, Grant JA, McLone DG. Presentation and management of hydromyelia in
children with Chiari type-II malformation. Pediatr Neurosurg. 1997 Feb;26(2):57-67. doi:
10.1159/000121166. PMID: 9419035.
Pubmed Web link: https://pubmed.ncbi.nlm.nih.gov/9419035/

40
Q

A surgical entry into the cavernous sinus is planned. The surgeon intends to enter by opening the infratrochlear (Parkinson’s) triangle. Which of the following nerves defines the lateral margin of this triangle?
Answers:
A. Trochlear nerve
B. Oculomotor nerve
C. Maxillary branch of the trigeminal nerve
D. Mandibular branch of the trigeminal nerve
E. Ophthalmic branch of trigeminal nerve

A

Ophthalmic branch of trigeminal nerve

Discussion:
Parkinson originally identified a triangle in the lateral wall of the cavernous sinus that would allow
for access to the intracavernous portion of the carotid artery to treat spontaneous carotidcavernous fistulas. The infratrochlear (Parkinson’s) triangle is bounded by cranial nerve (CN) V1
laterally, CN IV medially, while the base is formed by the line between the posterior limit of the
medial and lateral border. The infratrochlear triangle allows for access to CNs III, IV, and VI, as
well as the meningohypophyseal trunk and artery of the inferior cavernous sinus. The
supratrochlear triangle is bounded by CN III medially, CN IV laterally, and the base is formed by
the line between the posterior limit of the medial and lateral borders. This provides very narrow
access to the cavernous sinus.
References:
Microsurgical anatomy of the cavernous sinus: measurements of the triangles in and around it.
Skull Base. 2007. Nov;17(6):357-367.
Pubmed Web link: https://www.ncbi.nlm.nih.gov/pmc/articles/PMC2117623/
Intracranial anatomical triangles: A comprehensive illustrated review. Cureus. 2017. Oct
4;9(10):e1741.
Pubmed Web link: https://pubmed.ncbi.nlm.nih.gov/29218256/
Rhoton AL. The cavernous sinus, the cavernous venous plexus, and the carotid collar.
Neurosurgery, 2002. 51(suppl 4): S1-375-S1-410.
Pubmed Web link: https://academic.oup.com/neurosurgery/article/51/suppl_4/S1-375/2728121

41
Q

A 40-year-old man is evaluated because of a three-month history of loss of pain and temperature sensation in a cape-like distribution. He reports pain and weakness of the left shoulder. An x-ray film of the left shoulder is shown. Which of the following is the most likely diagnosis?
Answers:
A. Chari I malformation
B. Amyotrophic lateral Sclerosis
C. Pancoast Tumor
D. Transverse Myelitis
E. Subacute Combined Degeneration (SCD)

A

Chari I malformation

The correct answer is Chiari I malformation.
The pathogenesis of neuropathic arthropathy remains a controversial issue with 2 main theories:
the neurotraumatic and neurovascular theories.
X rays in this case demonstrate a left side Charcot Joint, a progressive destructive joint disease
associated with decreased joint sensation. Various etiologies include: syringomyelia (25%),
diabetes mellitus (0.16-2.5%) and tabes dorsalis (5-10%). Syringomyelia-associated Charcot’s joint
predominantly involves upper limbs (80%, shoulder joint followed by elbow) through interruption of
the crossing fibers of the lateral spinothalamic tracts around the central canal. Shoulder joint
symptoms frequently precede or overshadow neurological deficits. Loss of pain and temperature in
a classic cape distribution points to a central spinal cord pathology. A pathology resulting in a
progressive syrinx formation is to be suspected eg. Chiari I, post traumatic or tumor-associated
syrinx.
A Chiari I malformation would typically include low lying cerebellar tonsils and crowding of the
foramen magnum, impacting free CSF flow. This results in fluid accumulation in the central canal
forming a syrinx. Decompression of the cranio-cervical junction and restoration of free CSF flow
through a suboccipital craniectomy (with or without duroplasty) and C1 laminectomy would typically
lead to spontaneous resolution of a Chiari-associated syrinx.
SCD would be expected to impact the dorsal columns, in addition to peripheral neuropathy, while
ALS would present with pure upper and lower motor neuron deficits. Transverse Myelitis would
present in a more acute fashion with a generally regressive course and lead to involvement of
sensory and motor tracts. Pancoast tumor extension could compromise the lower trunk of the
brachial plexus in a unilateral fashion affecting the distal upper extremity. Although direct extension
is possible into the central canal, the presentation would include progressive myelopathy worse on
the ipsilateral side, rather than bilateral selective sensory loss in a cape distribution.
References:
Rickert MM, Cannon JG, Kirkpatrick JS. Neuropathic Arthropathy of the Shoulder: A Systematic
Review of Classifications and Treatments. JBJS Rev. 2019 Oct;7(10):e1.
Pubmed Web link: https://pubmed.ncbi.nlm.nih.gov/31663919/
Deng X, Wu L, Yang C, Xu Y. Neuropathic arthropathy caused by syringomyelia. J Neurosurg
Spine. 2013 Mar;18(3):303-9.
Pubmed Web link: https://pubmed.ncbi.nlm.nih.gov/23289508/

42
Q

A patient presents with a spinal cord injury. Motor examination shows 5/5 elbow flexion, 3/5 wrist extension, 0/5 elbow extension, and 0/5 hand function. There is no movement in the lower extremities. Sensation is preserved in the sacral dermatomes and in the right leg. The American
Spinal Injury Association Impairment Scale grade and motor level for this patient would be which of the following?
Answers:
A. ASIA B, C6 motor level
B. ASIA C, C7 motor level
C. ASIA C, C6 motor level
D. ASIA D, C7 motor level
E. ASIA A, C5 motor level

A

ASIA B, C6 motor level

ASIA scale:
A = Complete. No sensory or motor function is preserved in the sacral segments S4-S5.
B = Incomplete. Sensory but not motor function is preserved below the neurological level
and extends through the sacral segments S4-S5.
C = Incomplete. Motor function is preserved below the neurological level, and the majority
of key muscles below the neurological level have a muscle grade less than 3.
D = Incomplete. Motor function is preserved below the neurological level, and the majority
of key muscles below the neuro logical level have a muscle grade greater than or equal
to 3.
E = Normal. Sensory and motor function is normal.
Key muscle groups:
C5 - Elbow flexors (biceps, brachialis)
C6 - Wrist extensors (extensor carpi radialis longus and brevis)
C7 - Elbow extensors (triceps)
C8 - Finger flexors (flexor digitorum profundus) to the middle finger
TI - Small finger abductors (abductor digiti minimi)
L2- Hip flexors (iliopsoas)
L3 - Knee extensors (quadriceps)
L4- Ankle dorsiflexors (tibialis anterior)
L5 - Long toe extensors (extensor hallucis longus)
S1- Ankle plantarflexors (gastrocnemius, soleus)
The motor level is defined by the lower key muscle that has a grade of at least 3, providing the key
muscles represented by segments above that level are judged to be normal (4 or 5).
References:
American Spinal Injury Association. International Standards for Neurological Classifications of
Spinal Cord Injury. revised ed. Chicago, Ill: American Spinal Injury Association; 2000. 1-23.
Ditunno JF Jr, Young W, Donovan WH, Creasey G. The international standards booklet for
neurological and functional classification of spinal cord injury. American Spinal Injury Association.
Paraplegia. 1994 Feb;32(2):70-80. doi: 10.1038/sc.1994.13. PMID: 8015848.
Pubmed Web link: https://pubmed.ncbi.nlm.nih.gov/8015848/
Kirshblum S, Snider B, Rupp R, Read MS; International Standards Committee of ASIA and ISCoS.
Updates of the International Standards for Neurologic Classification of Spinal Cord Injury: 2015
and 2019. Phys Med Rehabil Clin N Am. 2020 Aug;31(3):319-330. doi: 10.1016/j.pmr.2020.03.005.
Epub 2020 Jun 3. PMID: 32624097.
Pubmed Web link: https://pubmed.ncbi.nlm.nih.gov/32624097/

43
Q

Which of the following pathways conveys light touch and proprioception?
Answers:
A. Medial longitudinal Fasiculus
B. Lateral Spinothalamic Tract
C. Reticulo-thalamic fibers
D. Pyramidal Tract
E. Internal Arcuate Fibers

A

Internal Arcuate Fibers

Discussion:
The dorsal column-medial lemniscus pathway is responsible for relaying proprioception and light
touch. Fibers from the first order neurons (located in the dorsal root ganglia) ascend uncrossed in
the spinal cord forming the Gracile and Cuneate tracts, synapsing with second order neurons in
the dorsal medulla. Fibers from the Gracile and Cuneate nuclei then cross forming the internal
arcuate fibers before ascending in the medial lemniscus, terminating on the thalamic nuclei (3rd
order neurons) that project to the sensorimotor cortex.
Pain and temperature are relayed through the lateral spinothalamic tract (direct pathway), or
through an indirect pathway through the reticulo-thalamic tract, where fibers from the second order
neurons (based in the dorsal horns) also cross the midline before terminating in the reticular
formation, that in turn project to the thalamus.
The Pyramidal cortico-spinal tract is an efferent pathway, while the Medial Longitudinal fasciculus
is a crossed tract within the brainstem connecting cranial nerves III, IV and VI.
References:
Kandel ER, Schwartz JH, Jessell TM, et al, eds. Principles of Neural Science. 5th ed. McGraw Hill,
New York, 2013:477.
Bradley WG, Daroff RB, Marsden CD, et al, eds. Neurology in Clinical Practice: Principles of
Diagnosis and Management, Vol. 1. 4th ed. London: Butterworth-Heinemann, 2005:360.

44
Q

An 81-year-old woman with a history of osteoporosis reports a one-week history of severe midback pain. She denies trauma or a recent fall. She is neurologically intact and able to ambulate, although she does require a walker as an ambulation aid due to pain. X-rays show a T11 compression fracture with a 20% loss of anterior column height. Which of the following is the most appropriate management at this time?
Answers:
A. Open T8-L2 instrumented fusion, T11 laminectomy
B. Percutaneous T9-L1 fixation
C. Trial of TLSO brace for 6-8 weeks, followed by T11 vertebroplasty / kyphoplasty if patient
fails conservative treatment
D. Posterior T11 corpectomy with cement reconstruction, and T9-L1 open fixation
E. Lateral retropleural T11 corpectomy with cage reconstruction and posterior percutaneous fixation T9-L1

A

Trial of TLSO brace for 6-8 weeks, followed by T11 vertebroplasty / kyphoplasty if patient fails conservative treatment

Discussion:
Individuals with a history of osteoporosis harbor an inherent risk of experiencing spontaneous
spinal compression fractures. These fractures can occur with very low impact activities or minor
trauma. The overwhelming majority of geriatric osteoporotic compression fractures are treated
conservatively. Unless there is severe kyphotic deformity and/or neurological deficit from
retropulsed bone fragments into the spinal canal, these fractures can safely be managed with a
trial of bracing, followed by vertebroplasty / kyphoplasty if the patient fails conservative treatment.
There is generally no role for upfront surgical intervention unless there is a substantial sagittal
plane deformity and/or neurological deficit. A lateral or posterior corpectomy would be an
extraordinarily aggressive upfront treatment for a geriatric compression fracture with 20% loss of
height.
References:
Jang HD, Kim EH, Lee JC, Choi SW, Kim K, Shin BJ. Current Concepts in the Management of
Osteoporotic Vertebral Fractures: A Narrative Review. Asian Spine J. 2020 Dec;14(6):898-909.
doi: 10.31616/asj.2020.0594. Epub 2020 Dec 22. PMID: 33373513; PMCID: PMC7788360
Pubmed Web link:
https://pubmed.ncbi.nlm.nih.gov/33373513/
Kweh BTS, Lee HQ, Tan T, Rutges J, Marion T, Tew KS, Bhalla V, Menon S, Oner FC, Fisher C,
Tee JW. The Role of Spinal Orthoses in Osteoporotic Vertebral Fractures of the Elderly Population
(Age 60 Years or Older): Systematic Review. Global Spine J. 2021 Jul;11(6):975-987. doi:
10.1177/2192568220948036. Epub 2020 Sep 29. PMID: 32990034; PMCID: PMC8258809
Pubmed Web link: https://pubmed.ncbi.nlm.nih.gov/32990034/

45
Q

A 74.8-kg (165-lb), 39-year-old man presents with an ASIA A spinal cord injury following a diving accident. The CT scans shown are obtained after hemodynamic stabilization. Attempts at closed reduction with tong traction up to 75 lbs have been unsuccessful. Which of the following is the
most appropriate management?
Answers:
A. Anterior C5 corpectomy + open facet reduction, posterior cervical decompression & fusion
B. Add further weight to traction, up to 125 lbs
C. Open facet reduction, posterior cervical decompression & fusion
D. Continue traction 75 lbs after adding further sedation / paralytics
E. Anterior C5 corpectomy

A

Open facet reduction, posterior cervical decompression & fusion

Discussion:
Bilateral traumatic jumped facets often are associated with a devastating neurological injury. The
primary goal of tong traction prior to surgical intervention is to provide an expeditious
decompression of the spinal canal via bilateral facet reduction. It is generally accepted to place
5-10 lbs of traction for the skull, and to add 5 lbs for each spinal level thereafter. 75 lbs of traction
in this case is rather generous. If no reduction is achieved at this stage with static traction, one
could certainly consider a manual reduction maneuver under fluoroscopy. If these maneuvers are
unable to achieve an expeditious spinal canal clearance by rapid facet reduction, the most
reasonable next step is to perform an open posterior reduction, with decompression and fusion.
While a C5 corpectomy would achieve clearance of the spinal canal, this would be an unorthodox
surgical intervention for this type of injury. A corpectomy alone would not likely yield a facet
reduction. Combining a corpectomy with a posterior decompression and fusion would be overly
aggressive, as canal decompression can be achieved with facet reduction alone. One of the
options not listed in the answer choices is a C4/5 ACDF. While this is certainly an option that would
spare multiple spinal levels from being fused, it is a less reliable method of rapid facet reduction
than direct posterior reduction.
References:
Rabinowitz RS, Eck JC, Harper CM Jr, Larson DR, Jimenez MA, Parisi JE, Friedman JA,
Yaszemski MJ, Currier BL. Urgent surgical decompression compared to methylprednisolone for the
treatment of acute spinal cord injury: a randomized prospective study in beagle dogs. Spine (Phila
Pa 1976). 2008 Oct 1;33(21):2260-8. doi: 10.1097/BRS.0b013e31818786db. PMID: 18827690
Pubmed Web link: https://pubmed.ncbi.nlm.nih.gov/18827690/
Wang MY, Prusmack CJ, Green BA, Gruen JP, Levi AD. Minimally invasive lateral mass screws in
the treatment of cervical facet dislocations: technical note. Neurosurgery. 2003 Feb;52(2):444-7;
discussion 447-8. doi: 10.1227/01.neu.0000043814.57539.59. PMID: 12535377
Pubmed Web link: https://pubmed.ncbi.nlm.nih.gov/12535377/

46
Q

A 53-year-old woman with a history of breast cancer is evaluated for lower extremity weakness that developed within the past 24 hours. An MR image is shown. Which of the following treatment strategies should be offered to the patient?
Answers:
A. Thoracic laminectomy and posterior epidural tumor debulking
B. High dose steroids and TLSO brace
C. Thoracic laminectomy and posterior epidural tumor debulking
D. Separation Surgery with instrumentation followed by stereotactic radiosurgery
E. Emergent stereotactic radiosurgery

A

Separation Surgery with instrumentation followed by stereotactic radiosurgery

Discussion:
Sudden onset lower extremity weakness in a patient with an oncologic history should raise
concerns of a compressive spinal metastatic deposit. In this case, the patient has a history of
breast cancer, which is “moderately sensitive” to conventional hypo-fractionated radiotherapy. The
MRI reveals circumferential spinal cord compression. While stereotactic radiosurgery is part of the
management in such a case, urgent spinal cord decompression and stabilization is in order as a
first-line treatment. Without clearing a several millimeter gap around the spinal cord (separation
surgery) first, the radiation oncologist will be forced to underdose the tumor in an effort to avoid
radiation-induced cord injury. Separation surgery entails performing a laminectomy, pediculectomy,
and sometimes facetectomy, to gain access to the ventral epidural space for anterior cord
decompression. Instrumentation is almost always necessary to avoid post-operative deformity. A
laminectomy alone does not allow for circumferential spinal cord decompression, and it may
potentiate a 3-column destabilization without supplemental instrumentation.
References:
Zuckerman SL, Laufer I, Sahgal A, Yamada YJ, Schmidt MH, Chou D, Shin JH, Kumar N, Sciubba
DM. When Less Is More: The indications for MIS Techniques and Separation Surgery in Metastatic
Spine Disease. Spine (Phila Pa 1976). 2016 Oct 15;41 Suppl 20(Suppl 20):S246-S253. doi:
10.1097/BRS.0000000000001824. PMID: 27753784; PMCID: PMC5551976
Pubmed Web link: https://pubmed.ncbi.nlm.nih.gov/27753784/
Patchell RA, Tibbs PA, Regine WF, et al. Direct decompressive surgical resection in the treatment
of spinal cord compression caused by metastatic cancer: a randomized trial. Lancet. 2005 Aug
20-26;366(9486):643-8
Pubmed Web link: https://pubmed.ncbi.nlm.nih.gov/16112300/

47
Q

In a patient who has spinal stenosis at L4-5, which of the following is most likely to cause compression of the L4 nerve root within the lateral recess?
Answers:
A. Pedicle
B. Inferior articular facet of the superior vertebra
C. Superior articular facet of the inferior vertebra
D. Vertebral body
E. Disc

A

Superior articular facet of the inferior vertebra

Discussion:
The lateral recess is bordered posteriorly by the ligamentum flavum and superior articular facet,
laterally by the pedicle, and anteriorly by the vertebral body and disk margin. The superior facet of
the lower vertebrae may cause nerve root pain of the level above in three ways (that is, the L5
superior facet causing L4 pain): (1) degenerative osteoarthritic changes in the facet, (2) facet
migration upward causing “facet impingement phenomena,” and (3) chondrocartilaginous material
encroachment from the facet joint compressing the nerve. Vertebral body osteophytes at the lateral
recess can cause lateral recess stenosis; however, it is not as common as the superior articular
facet impingement. Central disc herniation can cause impingement of the traversing nerve root
(L5) while foraminal disc herniation causes impingement of the exiting nerve root (L4). Pedicle and
inferior articular facet hypertrophy usually do not cause nerve root impingement.
References:
Colak A, Topuz K, Kutlay M, et al. A less invasive surgical approach in the lumbar lateral recess
stenosis: direct approach to the medial wall of the pedicle. Eur Spine J. 2008;17(12):1745-1751.
doi:10.1007/s00586-008-0801-z
Web link: https://www.ncbi.nlm.nih.gov/pmc/articles/PMC2587672/
Greenberg, M. S. (2020). Handbook of Neurosurgery. New York: Thieme Medical Publishers
Pubmed Web link: https://www.thieme.com/books-main/neurology/product/5411-handbook-ofneurosurgery

48
Q

A 36-year-old man who was the restrained driver in a motor vehicle collision presents to the emergency department. A CT scan is shown. Which of the following is the most likely classification of this patient’s spinal injury?
Answers:
A. AO Spine Classification – B1
B. AO Spine Classification – A1
C. AO Spine Classification – A4
D. AO Spine Classification – B3
E. AO Spine Classification – A2

A

AO Spine Classification – B1

Discussion:
The TLICS score and AO spine trauma classification are two commonly used systems to
characterize thoraco-lumbar spine fractures. In the AO spine classification system, there are three
main categories of injury mechanism: A (compression), B (trans-osseous tension band disruption),
and C (displacement/dislocation). The CT scan shown in the above case demonstrates a B1 injury,
which is a further sub-categorization of the trans-osseous fracture morphology. The vector of force
is horizontal to the orientation of the vertebral body. Trans-osseous fractures such as in this case
may initially be treated with a trial of bracing. Significant radiographic deformity and/or consistent
severe mechanical pain with standing upright posture may warrant surgical intervention.
AO Thoraco-Lumbar Spine Trauma Classification System:
A. Type A
A. A0 – Minor, non-structural fractures of the posterior elements
B. A1 – Simple, wedge compression fracture
C. A2 – Vertical split fracture (AKA Pincer fracture)
D. A3 – Incomplete burst fracture (only part of the vertebral body is affected)
E. A4 – Complete burst fracture (the entire vertebral body is affected)
B. Type B
A. B1 – Trans-osseous tension band disruption
B. B2 – Posterior tension band disruption
C. B3 – Trans-discal hyper-extension disruption
C. Type C
A. C – Displacement / Dislocation
References:
Koay J, Davis DD, Hogg JP. Chance Fractures. 2021 Aug 25. In: StatPearls [Internet]. Treasure
Island (FL): StatPearls Publishing; 2021 Jan–. PMID: 30725611.
Pubmed Web link: https://pubmed.ncbi.nlm.nih.gov/30725611/
Aly MM, Al-Shoaibi AM, Abduraba S, Alzahrani AJ, Eldawoody H. Traumatic low lumbar fractures:
How often MRI changes the fracture classification or clinical decision-making compared to CT
alone? Eur Spine J. 2021 Oct 8. doi: 10.1007/s00586-021-06987-x. Epub ahead of print. PMID:
34625851.
Pubmed Web link: https://pubmed.ncbi.nlm.nih.gov/34625851/

49
Q

A 30-year-old man is evaluated because of a four-day history of rapidly progressive bilateral lower extremity weakness, decreased sensation below the level of the umbilicus, and urinary retention. T2-weighted and T1-weighted MR images of the thoracic spine with contrast are shown.
Examination of the CSF, including neuromyelitis optica IgG, shows no abnormalities. Which of the following is the most appropriate initial step in management?
Answers:
A. Supportive care including maintenance of MAP above 80 mmHg
B. Interferon
C. High dose intravenous glucocorticoid therapy
D. T10-T12 laminectomy for spinal cord decompression
E. Excisional biopsy with subsequent radiation and chemotherapy

A

High dose intravenous glucocorticoid therapy

Discussion:
The most effective management for a first episode of transverse myelitis (TM) is high dose
intravenous glucocorticoid therapy. In a patient with rapid onset of motor, autonomic, and sensory
dysfunction localizing to a spinal cord lesion, TM must be considered at the top of the differential.
TM lesions are heterogeneous and can involve a focal area of enhancement in 50% of cases.
When TM is suspected, additional workup to rule out associated disorders such as neuromyelitis
optica and multiple sclerosis is mandatory. While a spinal cord malignancy may mimic the
radiographic appearance of TM, symptoms tend to arise over a much greater timespan. There is
no surgical intervention for TM. Supportive care with maintenance of MAP > 80 mmHg is
appropriate for instances of traumatic spinal cord injury but has not been shown to be beneficial for
acute TM. Interferon-β (IFN-β) is a treatment option for MS and has well-documented benefit for
patients with the relapsing and secondary progressive variants.
References:
Jacobs LD, Beck RW, Simon JH, Kinkel RP, Brownscheidle CM, Murray TJ, Simonian NA, Slasor
PJ, Sandrock AW. Intramuscular interferon beta-1a therapy initiated during a first demyelinating
event in multiple sclerosis. CHAMPS Study Group. N Engl J Med. 2000 Sep 28;343(13):898-904.
doi: 10.1056/NEJM200009283431301. PMID: 11006365.
Pubmed Web link: https://pubmed.ncbi.nlm.nih.gov/11006365/
Choi KH, Lee KS, Chung SO, Park JM, Kim YJ, Kim HS, Shinn KS. Idiopathic transverse myelitis:
MR characteristics. AJNR Am J Neuroradiol. 1996 Jun-Jul;17(6):1151-60. PMID: 8791931; PMCID:
PMC8338601.
Pubmed Web link: https://pubmed.ncbi.nlm.nih.gov/8791931/
Scott TF, Frohman EM, De Seze J, Gronseth GS, Weinshenker BG; Therapeutics and Technology
Assessment Subcommittee of American Academy of Neurology. Evidence-based guideline: clinical
evaluation and treatment of transverse myelitis: report of the Therapeutics and Technology
Assessment Subcommittee of the American Academy of Neurology. Neurology. 2011 Dec
13;77(24):2128-34. doi: 10.1212/WNL.0b013e31823dc535. Epub 2011 Dec 7. PMID: 22156988.
Pubmed Web link: https://pubmed.ncbi.nlm.nih.gov/22156988/

50
Q

Which of the following are the key characteristics of an anterior spinal cord syndrome following a spinal cord injury?
Answers:
A. Loss of motor, pain, and temperature below the level of the lesion
B. Upper extremity greater than lower extremity loss of sensation
C. Loss of motor, pain, and temperature below the level of the lesion
D. Upper extremity greater than lower extremity loss of sensation
E. loss of pain, temperature, and vibration sensation below the injury

A

Loss of motor, pain, and temperature below the level of the lesion

Discussion:
ue:
Anterior cord syndrome can occur traumatically as a result of anterior cord compression or
occlusion of the anterior spinal artery. It presents with loss of motor (corticospinal tracts), pain and
temperature (spinothalamic tracts) below the level of the lesion, with preservation of dorsal column
functions.
Central cord syndrome is usually a result of acute hyperextension injury in older patients in the
presence of osteophytic spurs and redundant ligamentum flavum. The injury most often occurs
with a blow to the upper face or forehead. The syndrome is thought to result from edema within the
vascular watershed zone in the central region of the cord; cervical long tract fibers are located
more medially than lower extremity fibers. Consequently, there exists significantly greater motor
deficit in the upper extremities than the lower. Sensory disturbance is more variable and is present
below the level of the lesion.
Posterior cord syndrome, which is relatively rare, is due to selective injury or ischemia to
the dorsal columns, and produces sensory ataxia, decreased vibration, and light touch. Mild
weakness of the upper extremities can also occur.
Brown-Sequard syndrome is also referred to as spinal cord hemisection and is usually secondary
to penetrating rather than blunt trauma. The findings include ipsilateral motor paralysis, loss of
dorsal column function below the level of the lesion, and contralateral loss of pain and temperature
sensation 1-2 segments below the lesion. Brown-Sequard syndrome is also referred to as lateral
cord syndrome.
References:
Yadav N, Pendharkar H, Kulkarni GB. Spinal Cord Infarction: Clinical and Radiological Features. J
Stroke Cerebrovasc Dis. 2018 Oct;27(10):2810-2821. doi:
10.1016/j.jstrokecerebrovasdis.2018.06.008. Epub 2018 Aug 6. PMID: 30093205.
Pubmed Web link: https://pubmed.ncbi.nlm.nih.gov/30093205/
Taylor TR, Dineen R, White B, Jaspan T. The thoracic anterior spinal cord adhesion syndrome. Br
J Radiol. 2012 Jun;85(1014):e123-9. doi: 10.1259/bjr/81458631. PMID: 22665931; PMCID:
PMC3474120.
Pubmed Web link: https://pubmed.ncbi.nlm.nih.gov/22665931/

51
Q

Which of the following methods of spinal fixation requires the largest force to disrupt the boneimplant interface in the non-osteoporotic spine?
Answers:
A. Sublaminar wires
B. Sublaminar cables
C. Transverse process hooks
D. Laminar hooks
E. Pedicle screws

A

Pedicle screws

Discussion:
Pedicle screws are the most biomechanically sound devices of the options listed in terms of rate of
implant failure at the bone-device interface in the non-osteoporotic spine. Historically, spine
surgeons used hooks (sublaminar or TP) in addition to wires/cables, but these older methods have
been shown to be inferior to screws in tests of pullout strength. That said, some surgeons will use
hooks at the upper instrumented vertebrae (UIV) level in a large deformity construct to create a
“soft landing” and reduce the incidence of proximal junctional kyphosis (PJK). In patients with
normal bone density, pedicle screws demonstrate superior biomechanical properties regarding
pullout forces. Sublaminar hooks are next strongest, followed by sublaminar wires/cables. Cables
are several wires in one bundle.
There are newer lumbar screws called cortical bone trajectory (CBT) screws that start medially and
take advantage of the dorsal cortical bone in the lamina and pars. The initial insertion point of CBT
screws is the junction of the superior articular process and pars, which allows for minimal soft
tissue dissection and may reduce the risk of neurovascular injury. CBT screws involve a caudal to
cranial path and medial-to-lateral path with the objective of maximizing cortical bone contact
through the pedicle to the vertebral body and minimizing the risk of instrumentation failure.
Whether a true pedicle screw or a CBT screw is stronger remains an evolving debate. Pedicle
screws are longer and go farther into the body, whereas CBT screws are shorter and have less
purchase in the anterior column. The main advantage to CBT screws it not having to dissect
laterally to the transverse processes. Most studies show no difference in long term outcome, but
several studies have shown that CBT was associated with less blood loss, less operative time, and
shorter hospital stay compared to pedicle screws (PMID: 31415372). A small number of
biomechanical studies show that CBT screws have similar pullout strengths to pedicle screws
(PMID: 27176113).
References:
Hackenberg L, Link T, Liljenqvist U. Axial and tangential fixation strength of pedicle screws versus
hooks in the thoracic spine in relation to bone mineral density. Spine (Philadelphia PA 1976). 2002
May 1; 27(9):937-42.2.
Pubmed Web link: https://pubmed.ncbi.nlm.nih.gov/11979165/
Hitchon PW, Brenton MD, Black AG, et al. In vitro biomechanical comparison of pedicle screws,
sublaminar hooks, and sublaminar cables. J Neurosurgery. 2003 July (1 Supplement):104-9.
Pubmed Web link: https://pubmed.ncbi.nlm.nih.gov/12859069/
Lee GW, Son JH, Ahn MW, Kim HJ, Yeom J. The comparison of pedicle screw and cortical screw
in posterior lumbar interbody fusion: a prospective randomized noninferiority trial. Spine J. 2015.
2015 Feb 26. PMID: 25728553
Pubmed Web link: https://pubmed.ncbi.nlm.nih.gov/25728553/

52
Q

A 28-year-old man presents after a diving injury with a C5 fracture and severe retropulsion of bone fragments. His American Spinal Injury Association (ASIA) score is B at the level of C5. Which of the following findings is most consistent with this score?
Answers:
A. 5/5 motor strength in biceps & deltoids with preserved peri-rectal sensation
B. 5/5 motor strength in biceps & deltoids without preserved peri-rectal sensation
C. 5/5 motor strength in wrist extensors with preserved peri-rectal sensation
D. No motor activity in any muscle group of the bilateral upper extremities with preserved peri-rectal sensation
E. 5/5 motor strength in hand intrinsics with preserved peri-rectal sensation

A

5/5 motor strength in biceps & deltoids with preserved peri-rectal sensation

Discussion:
Spinal cord injuries are classified by the last normal spinal segment in reference to motor activity,
and the degree to which there is preserved motor and sensory function below the level of the
injury. In this particular case, one would expect that the patient would have normal strength in his
biceps & deltoids (C5 innervation), but no appreciable motor activity below the level of injury. It
should be noted that both C5 & C6 contribute to biceps function. As a result, the biceps is spared
in this scenario, while wrist extensor function (C6) would be affected.
Additionally, “sacral sparing,” or preserved peri-rectal sensation would be appreciated on
neurological assessment, classifying this patient as an ASIA B.
ASIA A – No motor or sensory function is preserved below the neurological level of injury
ASIA B – No motor function is preserved below the neurological level of injury, but sensory function
is spared in the peri-anal region (S4/5 segments)
ASIA C – Motor function is preserved below the neurological level of injury, but half of the key
muscle groups have a grade < 3
ASIA D – Motor function is preserved below the neurological level of injury, and at least half of the
key muscle groups have a grade > 3
ASIA E – Motor and sensory function is normal
References:
Kirshblum SC, Burns SP, Biering-Sorensen F, Donovan W, Graves DE, Jha A, et al. International
standards for neurological classification of spinal cord injury (revised 2011). J Spinal Cord Med.
2011 Nov;34(6):535-46.
Pubmed Web link: https://pubmed.ncbi.nlm.nih.gov/22330108/
Kirshblum S, Snider B, Eren F, Guest J. Characterizing Natural Recovery after Traumatic Spinal
Cord Injury. J Neurotrauma. 2021 May 1;38(9):1267-1284. doi: 10.1089/neu.2020.7473. Epub
2021 Jan 22. PMID: 33339474; PMCID: PMC8080912
Pubmed Web link: https://pubmed.ncbi.nlm.nih.gov/33339474/

53
Q

A 65-year-old woman presents for evaluation of progressive cervical myelopathy. Imaging shows multilevel stenosis at C3-4, C4-5, and C5-6. Which of the following factors would be an absolute contraindication for a dorsal cervical decompression procedure via laminoplasty?
Answers:
A. Single level stenosis
B. Cervical lordosis
C. Cervical myelopathy
D. Cervical kyphosis
E. Neck pain

A

Cervical kyphosis

Discussion:
is a contraindication for a dorsal cervical decompression via laminoplasty. Laminoplasty does not
address spinal cord compression in a kyphotic cervical spine. This does not obtain sufficient dorsal
migration of the cord and can contribute to instability and worsening of cervical kyphosis. Cervical
lordosis is preferred when considering a patient for cervical laminoplasty as this facilitates a high
likelihood of dorsal cord migration. Neck pain is not an indication for laminoplasty, but it is not an
absolute contraindication. Single level stenosis is not recommended when considering
laminoplasty, as more levels facilitates a greater yield of cord migration from anterior structures.
Cervical myelopathy is a clinical finding that may improve after cervical laminoplasty is completed
on the proper patient.
References:
Mitsunaga LK, Klineberg EO, Gupta MC. Laminoplasty techniques for the treatment of multilevel
cervical stenosis. Adv Orthop. 2012;2012:307916.
Pubmed Web link: https://www-ncbi-nlm-nih-gov.geihsl.idm.oclc.org/pmc/articles/PMC3310284/
Suda, Kota MD*
; Abumi, Kuniyoshi MD*
; Ito, Manabu MD*
; Shono, Yasuhiro MD*
; Kaneda, Kiyoshi
MD*
; Fujiya, Masanori MD† Local Kyphosis Reduces Surgical Outcomes of Expansive Open-Door
Laminoplasty for Cervical Spondylotic Myelopathy, Spine: June 15, 2003 - Volume 28 - Issue 12 - p
1258-1262 doi: 10.1097/01.BRS.0000065487.82469.D9

54
Q

The lesion depicted in the MR images shown is most likely to result in which of the following?
Answers:
A. Weakness in the right deltoid
B. Positive Hoffman’s sign on the right side
C. Positive Hoffman’s sign on the left side
D. Wrist drop on the left side
E. Numbness/tingling in the 4th and 5th digits on the left side

A

Wrist drop on the left side

Discussion:
The lesion depicted in the MRI is a C6-7 disc herniation. The image shown is a T2 MRI, sagittal
and axial views, of the cervical spine. This reveals a C6-7 disc herniation with left sided foraminal
stenosis and exiting nerve root compression. This left sided C6-7 disc herniation will affect the left
C7 exiting nerve root resulting in a left C7 radiculopathy. This would present with weakness in
forearm extension, wrist drop, diminished triceps reflexes and sensory changes in the 2nd and 3rd
digits. A C5 radiculopathy would result in deltoid weakness. A C8 radiculopathy may result in
hand intrinsics weakness and 4th and 5th digit sensory issues. Hoffman’s sign is suggestive of a
spinal cord issue, most specifically with the corticospinal tract. This disc is not compressing the
spinal cord.
References:
Greenberg MS, 9th ed. Handbook of Neurosurgery. New York, NY: Thieme, 2020.
Patton HD, ed. Introduction to Basic Neurology. Philadelphia, PA: WB Saunders; 1976.

55
Q

Which of the following conditions is most likely in a child with intraspinal arachnoid cysts?
Answers:
A. Chiari Malformation
B. Neural Tube defects
C. Scoliosis
D. Intracranial Arachnoid cysts
E. Hydrocephalus

A

Neural Tube defects

Discussion:
Neural tube defects such as myelomeningocele, tethered cord, and diastematomyelia are most
commonly associated with arachnoid cysts. Though other answers described above can be
associated with neural tube defects, none of them are directly associated with spinal arachnoid
cysts. The incidence of having an intracranial arachnoid cyst does not increase in patients with
spinal arachnoid cysts. Hydrocephalus, Chiari malformation, and scoliosis are all associated with
myelomeningoceles, though not directly with spinal arachnoid cysts.
References:
Rabb CH, McComb JG, Raffel C, Kennedy JG. Spinal arachnoid cysts in the pediatric age group:
an association with neural tube defects. J Neurosurg. 1992 Sep;77(3):369-72. doi:
10.3171/jns.1992.77.3.0369. PMID: 1506883.
Pubmed Web link: https://pubmed.ncbi.nlm.nih.gov/1506883/
Patankar AP. Spinal intradural arachnoid cyst associated with diastematomyelia in an adult: a case
report and review of literature. Br J Neurosurg. 2019 Sep 17:1-3. doi:
10.1080/02688697.2019.1666087. Epub ahead of print. PMID: 31526134.
Pubmed Web link: https://pubmed.ncbi.nlm.nih.gov/31526134/

56
Q

A 35-year-old woman is evaluated for thoracolumbar scoliosis. She has two curves: a dextroscoliosis in the lumbar spine 40 degrees and a levoscoliosis in the thoracic spine 33 degrees. Side-bending x-ray films show reduction of the lumbar curve to 25 degrees, a reduction of the thoracic curve to 10 degrees, and her head centered over her pelvis. Which of the following
best describes this patient’s scoliosis?
Answers:
A. Adolescent idiopathic scoliosis
B. Flexible
C. Rigid
D. Functional
E. Fused

A

Flexible

Discussion:
The patient described here has scoliosis that is flexible, since both her curves reduce down with
bending x-rays. When talking about idiopathic scoliosis – either adolescent idiopathic scoliosis
(AIS, <18yo) or adult idiopathic scoliosis (AdIS, >18yo), we divide the spine into 4 segments in the
coronal plane, since these are most often a coronal problem rather than sagittal problem (though
the sagittal plane is very important): 1) proximal thoracic (PT), 2) main thoracic (MT), 3)
thoracolumbar/lumbar (TL/L), and 4) lumbosacral fractional curve. It appears this patient has a
commonly seen pattern in AIS and AdIS with a right MT curve and compensatory TL/L curve to the
left. The classification of this curve can be drawn from the Lenke Classification that comes from
the AIS literature, with Type 1 to Type 6. There are two aspects of scoliosis that are crucial when
making a treatment plan, whether it be operative or nonoperative. First, you must see how flexible
or rigid the curves are. This is classically done with bending films, but also supine films are a great
way to see how much the curves change when gravity is removed. Bending films are becoming
less popular because they are highly dependent on patient effort and radiology technician
instructions, which are both highly variable. Comparing upright scoliosis films to supine films – PA
and lateral – can be helpful to see how flexible a curve is. A flexible curve may only need posterior
column osteotomies, but if a spine is fixed or rigid, a three-column osteotomy may be necessary.
Second, it is important to see the global alignment, i.e. where the head or C7 is centered over the
pelvis. The SVA (sagittal vertical axis) is measured to assess if C7 or the external auditory canal
(EAC) is centered over the back of the S1 endplate. Generally, a number <5cm is the goal. The
CVA (coronal vertical axis) is the same measurement in the coronal plane, where C7 or the skull is
compared to the central sacral vertical line (CSVL ). Generally, a CVA <3cm is aimed for, but this is
not an agreed upon number and remains a very active area in the literature.
References:
Heary R, Alpert T. Spinal Deformities: The Essentials. Thieme; 2007: 45-59.
Lin JD, Osorio JA, Baum GR, Menger RP, Reid PC, Dyrszka MD, Amorosa LF, Sardar ZM,
Mandigo CE, Angevine PD, Kelly MP, Cerpa M, Lenke LG. A new modular radiographic
classification of adult idiopathic scoliosis as an extension of the Lenke classification of adolescent
idiopathic scoliosis. Spine Deform. 2021 Jan;9(1):175-183. PMID: 32748229
Pubmed Web link: https://pubmed-ncbi-nlm-nih-gov.proxy.library.vanderbilt.edu/32748229/

57
Q

A 27-year-old construction worker has sudden onset of severe headache while doing heavy lifting at work. His symptoms improve when he is supine. On initial evaluation at a local emergency department, neurological examination, CT scan of the head, and spinal fluid analysis show no
abnormalities. A few days later, he presents with persistent positional headaches, nausea, and new left abducens nerve palsy. Non contrast T1, T2, and contrast T1-weighted axial MR images are shown. Which of the following is the most likely diagnosis?
Answers:
A. Complex Migraine
B. Perineural Cyst
C. Aneurysmal Subarachnoid hemorrhage
D. Postural Tachycardia Syndrome
E. Pseudotumor Cerebri

A

Perineural Cyst

Discussion:
The correct answer is perineural cyst. Perineural cysts are arachnoid diverticula originating from
the nerve root sheath. They are often incidental findings on imaging, though may be large enough
to exert mechanical pressure on adjacent roots. Occasionally, these cysts may rupture, leading to
cerebrospinal fluid (CSF) leak, which can cause intracranial hypotension, which this patient
presents with. Associated symptoms include positional headaches that improve in the supine
position, nausea/emesis, and cranial nerve palsy. Abducens palsy is the most common associated
cranial nerve palsy, due to its long length and course over the tentorial edge.
Spine MRI may demonstrate epidural CSF collections associated with an arachnoid diverticulae,
and distended epidural venous plexus. CT myelogram is useful for evaluation of these cysts, as it
can demonstrate whether the cyst communicates with the subarachnoid space, which can
influence cyst treatment planning. Treatment options include epidural blood patch, cyst
fenestration, and cyst resection.
Aneurysmal subarachnoid hemorrhage (SAH) induced headaches would not resolve in the supine
position, nor Pseudotumor cerebri headache. The positional nature of the headache in this patient
as well as imaging characteristics rule out a migraine headache. Postural tachycardia syndrome
can present as an orthostatic headache, but in such cases, there is an obvious postural drop in
blood pressure, and absence of abducens palsy with associated intracranial MRI findings.
References:
Upadhyaya P, Ailani J. A Review of Spontaneous Intracranial Hypotension. Curr Neurol Neurosci
Rep. 2019 Mar 19;19(5):22.
Pubmed Web link: https://www.ncbi.nlm.nih.gov/pubmed/30888542
Starling A, Hernandez F, Hoxworth JM et al. Trentman T, Halker R, Vargas BB, Hastriter E, Dodick
D. Sensitivity of MRI of the spine compared with CT myelography in orthostatic headache with
CSF leak. Neurology. 2013 Nov 12;81(20):1789-92.
Pubmed Web link: https://www.ncbi.nlm.nih.gov/pubmed/24107860

58
Q

The parasympathetic innervation of the sublingual gland is provided by which of the following structures?
Answers:
A. Greater petrosal nerve
B. Nucleus tractus solitarious
C. Vagus nerve
D. Glossopharyngeal nerve
E. Chorda tympani

A

Chorda tympani

Discussion:
The chorda tympani carries the preganglionic parasympathetic nerve fibers, which provide
innervation to the submandibular and sublingual glands. The parasympathetic innervation of the
lacrimal gland is complex, with the preganglionic parasympathetic neurons located in the superior
salivatory nucleus, which are then projected along the greater petrosal branch of the facial nerve.
The nucleus tractus solitarius is the destination of afferent taste fibers, which also run along the
chorda tympani.
References:
Regulation of salivary gland function by autonomic nerves. Auton Neurosci. 2007. Apr
30;133(1):3-18.
Pubmed Web link: https://pubmed.ncbi.nlm.nih.gov/17157080/
Effects of the chorda tympani damage on submandibular glands: scintigraphic changes. J Laryngol
Otol. 2004. Feb;118(2):102-105.
Pubmed Web link: https://pubmed.ncbi.nlm.nih.gov/14979945/

59
Q

A 45-year-old man is evaluated because of pain in the right radial forearm and thumb, as well as weakness of forearm flexion. Which of the following is the most likely diagnosis?
Answers:
A. C5 radiculopathy from C5-6 disc herniation compressing C5 nerve root
B. C6 radiculopathy from C5-6 disc herniation compressing C6 nerve root
C. C6 radiculopathy from C6-7 disc herniation compressing C6 nerve root
D. C7 radiculopathy from C6-7 disc herniation compressing C7 nerve root
E. C5 radiculopathy from C4-5 disc herniation compressing C5 nerve root

A

C6 radiculopathy from C5-6 disc herniation compressing C6 nerve root

Discussion:
The most likely diagnosis is C6 radiculopathy from a C5-6 disc herniation compressing the C6 nerve root. A disc herniation at the level of C5-6 will affect the C6 nerve root. A C5-6 disc herniation accounts for about 19% of cervical disc herniations. A C6 radiculopathy will result in weakness in forearm flexion, sensory issues in the upper arm, thumb and radial forearm, and diminished biceps and brachioradialis reflexes. A C6 nerve root would not get affected with a C6-7
disc herniation. A C5 nerve root would not get affected with a C5-6 disc herniation. C5 radiculopathy affects the deltoids. A C7 radiculopathy results in triceps weakness.
References:
Greenberg M, ed. Handbook of Neurosurgery. 9th ed. Thieme, 2020.
MacNab I. Cervical spondylosis. J Clinical Orthop Relat Res. 1975; 109:69-77.

60
Q

A 21-year-old-man is evaluated for a progressive history of difficulty walking. Position, vibration, and deep tendon reflexes are absent in his lower extremities. He has up-going toes bilaterally, and a gait that is broad-based and grossly impaired. His uncle had similar symptoms, developed diabetes and deafness, and died at age 55 years due to cardiomyopathy. Which of the following is the most likely diagnosis?
Answers:
A. Wilson’s disease
B. Posterior fossa neoplasms
C. Friedreich’s ataxia
D. Multiple sclerosis
E. Huntington’s chorea

A

Friedreich’s ataxia

Discussion:
This man has Friedreich’s Ataxia, an autosomal recessive condition caused by an unstable GAA
trinucleotide repeat in a gene for a mitochondrial protein named frataxin. Patients develop
symptoms of gait instability and lower extremity sensory disturbance in adolescence. Symptoms
progress, eventually involving the arms. 50% have skeletal deformities, including pes cavus and
scoliosis. 60% can have a hypertrophic cardiomypopathy. Other symptoms can include deafness,
optic atrophy, and diabetes mellitus. Pathologic changes are characterized by demyelination and
degeneration of the posterior columns, spinocerebellar, and pyramidal tracts in the spinal cord.
Similar changes are seen in the cerebellar dentate nuclei, dorsal root ganglia, and peripheral
nerves (loss of large myelinated fibers). Diagnosis is by genetic confirmation of the mutation on
chromosome 9. Care is supportive. Life expectancy is typically 35-40 years from onset.
Huntington’s disease (HD), also known as Huntington’s chorea, is a neurodegenerative
disease that is mostly inherited. The earliest symptoms are often subtle problems with mood or
mental abilities. A general lack of coordination and an unsteady gait often follow. It is a basal
ganglia disease causing a hyperkinetic movement disorder known as chorea. As the disease
advances, uncoordinated, involuntary body movements of chorea become more apparent.
Physical abilities gradually worsen until coordinated movement becomes difficult and the person is
unable to talk. Mental abilities generally decline into dementia. The specific symptoms vary
somewhat between people. Symptoms usually begin between 30 and 50 years of age but can start
at any age.
Wilson’s disease is a genetic disorder in which excess copper builds up in the body. Symptoms are
typically related to the brain and liver. Liver-related symptoms include vomiting, weakness, fluid
buildup in the abdomen, swelling of the legs, yellowish skin and itchiness. Brain-related symptoms
include tremors, muscle stiffness, trouble speaking, personality changes, anxiety, and psychosis. Posterior fossa tumor is a type of brain tumor located in or near the bottom of the skull. Symptoms
occur very early with posterior fossa tumors and may include drowsiness, headache, imbalance,
uncoordinated walk (ataxia), nausea, and vomiting. Multiple sclerosis (MS), also known as encephalomyelitis disseminata, is the most
common demyelinating disease, in which the insulating covering of nerve cells in
the brain and spinal cord (myelin) is damaged. This damage disrupts the ability of parts of the
nervous system to transmit signals, resulting in a range of signs and symptoms, including
physical, mental, and sometimes psychiatric problems. Specific symptoms can include double
vision, blindness, muscle weakness, and trouble with sensation or coordination. MS takes several
forms, with new symptoms either occurring in isolated attacks (relapsing forms) or building up over
time (progressive forms). Between attacks, symptoms may disappear completely, although
permanent neurological problems often remain, especially as the disease advances.
References:
Pandolfo M. Friedreich ataxia. Arch Neurol. 2008 Oct;65(10):1296-303. doi:
10.1001/archneur.65.10.1296. PMID: 18852343.
Pubmed Web link: https://pubmed.ncbi.nlm.nih.gov/18852343/
Delatycki MB. Evaluating the progression of Friedreich ataxia and its treatment. J Neurol. 2009
Mar;256 Suppl 1:36-41. doi: 10.1007/s00415-009-1007-y. PMID: 19283349.
Pubmed Web link: https://pubmed.ncbi.nlm.nih.gov/19283349/

61
Q

A 23-year-old man with cerebral palsy and severe spasticity receives an intrathecal pump after developing dose-limiting toxicities to oral baclofen. Three years later, the catheter is damaged during a planned pump change, and an entirely new system is implanted. The following morning, the patient is observed to be hypothermic, hypotensive, and lethargic. Today, scheduled
postoperative x-rays, obtained three hours before, are shown. Which of the following is the most appropriate next step in management?
Answers:
A. Empty the pump reservoir
B. Intubation
C. Nasogastric tube placement, oral Baclofen
D. Physostigmine 1 mg IV
E. Lumbar Drain with intrathecal Baclofen infusion

A

Intubation

Discussion:
The correct answer is Intubation.
The patient in this scenario demonstrates signs of baclofen overdose, which is characterized by
hypotension, flaccid muscle tone, depressed mental status, and/or respiratory depression.
Baclofen withdrawal is primarily characterized by tachycardia, hypertension, fever, hypertonia,
rhabdomyolysis, and possibly seizures. Post-operative x-rays indicate an intact tubing system.
First-line management of baclofen overdose patients with signs of altered mental status and
cardiopulmonary compromise includes airway protection, breathing assistance, and cardiac
resuscitation (ABC). There is no baclofen specific antidote; however, there have been case reports
demonstrating the possible utility of Physostigmine IV for symptomatic relief. To reduce the
concentration of baclofen in the cerebrospinal fluid (CSF), one can: 1) empty the pump reservoir,
and/or 2) aspirate 40 cc of CSF through the pump port or via lumbar puncture. Giving the patient
oral or intrathecal baclofen would not be appropriate in the setting of baclofen overdose.
References:
Saulino M, Anderson DJ, Doble J, et al. Best Practices for Intrathecal Baclofen Therapy:
Troubleshooting. Neuromodulation. 2016 Aug;19(6):632-41.
Romito JW, Turner ER, Rosener JA, Coldiron L, Udipi A, Nohrn L, Tausiani J, Romito BT. Baclofen
therapeutics, toxicity, and withdrawal: A narrative review. SAGE Open Med. 2021 Jun
3;9:20503121211022197. doi: 10.1177/20503121211022197. PMID: 34158937; PMCID:
PMC8182184

62
Q

Parasympathetic innervation of the bladder and the afferent fibers arising from stretch receptors in the bladder wall are both contained in which of the following structures?
Answers:
A. T10-L2 region of the spinal cord
B. S2-4 dorsal root ganglia
C. Rostral pons
D. Inferior mesenteric ganglion
E. S2-4 region of the spinal cord

A

S2-4 region of the spinal cord

Discussion:
Bladder filling and emptying depends primarily on a parasympathetic reflex arc. This arc originates
with stretch mechanoreceptors within the bladder wall that transmit visceral information via
parasympathetic afferents to the spinal cord, specifically within the S2-4 segment, termed the
center for micturition. Parasympathetic efferents are then activated in this same area, and
information is relayed back to the bladder detrusor muscle via the pelvic plexus. In addition to the
spinal cord, parasympathetic afferent information is transmitted to higher autonomic centers in the
brainstem including the pontomesencephalic reticular formation micturition center which project to
various central coordinating centers for bladder function in the brainstem tegmentum, as well as
the frontal lobe and insula. The sympathetic innervation of the bladder originates in the
thoracolumbar junction spinal cord segments (T10-L2), and the preganglionic axons run to
sympathetic neurons in the inferior mesenteric ganglion and the ganglia of the pelvic plexus.
References:
Purves D, Augustine GJ, Fitzpatrick D, et al., editors. Neuroscience. 2nd Edition.
Sunderland:Sinauer Associates, 2001. Autonomic regulation of the bladder.
Pubmed Web link: https://www.ncbi.nlm.nih.gov/books/NBK10886/
The central autonomic network and regulation of bladder function. Front Neurosci. 2019. Jun
13;13:535.
Pubmed Web link: https://pubmed.ncbi.nlm.nih.gov/31263396/

63
Q

Voluntary contraction of the external bladder sphincter is innervated by which of the follow structures?
Answers:
A. Parasympathetic preganglionic neurons S2-4
B. Hypogastric nerves
C. Inferior mesenteric ganglia
D. Pudendal nerve
E. Ganglia of the pelvic plexus

A

Pudendal nerve

Discussion:
The innervation of the urethral sphincter is from both the somatic and the autonomic nervous
systems. Urination is prevented by the voluntary motor innervation of the external urinary
sphincter, which is a striated muscle that is innervated by the pudendal nerve from the S2-S4
nerve roots, coming from Onuf’s nucleus. The autonomic nervous system controls the innervation
of the internal urinary sphincter, which is made of smooth muscle. The sympathetic innervation
from the T10-L2 spinal cord sends preganglionic axons to sympathetic neurons in the inferior
mesenteric ganglion and the ganglia of the pelvic plexus. The postganglionic fibers travel via the
hypogastric and pelvic nerves to the bladder. Sympathetic activity results in closing of the internal
urinary sphincter, while parasympathetic activity causes the bladder to contract and the internal
sphincter to relax and open.
References:
Clinical and functional anatomy of the urethral sphincter. Int Neurourol J. 2012. Sep;16(3):102-106.
Pubmed Web link:
https://www.ncbi.nlm.nih.gov/pmc/articles/PMC3469827/
Continence and micturition: an anatomical basis. Clin Anat. 2014. Nov;27(8):1275-83.
Pubmed Web link: https://pubmed.ncbi.nlm.nih.gov/24615792/

64
Q

A 19-year-old woman who is a complete C5 quadriplegic after sustaining a C6 burst fracture with significant canal compromise and spinal cord injury, undergoes C6 corpectomy with cage and cervical plate and C4-7 posterior instrumented fusion. While at the rehabilitation center, she develops drainage from her neck. A laryngoscopic image is shown. Which of the following is the most likely diagnosis?
Answers:
A. Pseudomeningocele
B. Pharyngocutaneous Fistula
C. Arytenoid Dislocation
D. Zenker’s diverticulum
E. Laryngocele

A

Pharyngocutaneous Fistula

Discussion:
Drainage from the neck following a prior anterior cervical spine surgery should immediately raise
suspicion of injury to the esophagus or pharynx. Saliva leaks into the surrounding neck soft tissues
and out of the skin. In the chronic setting, this may be due to erosion of an anterior cervical plate or
screw into the overlying esophagus. There is a significantly increased risk of infection with injury to
the esophagus, which can occur in up to 0.1% of anterior cervical spine surgeries. Direct
visualization with esophagoscopy in addition to imaging is recommended, as is
otorhinolaryngology consultation to help with repair of the fistula. Other risks of anterior cervical
spine surgery include cerebrospinal fluid (CSF) leak (0.5%-1.7%) and wound infection (0.1-1.6%).
While an acute pseudomeningocele may cause swelling, wound drainage and low-pressure
headaches, there would be no associated violation to the pharyngeal or esophageal wall.
Arytenoid dislocation, while quite uncommon, would present with hoarseness and vocal cord
asymmetry – mild cases reduce spontaneously, while more extensive cases require laryngoscopic
intervention. Laryngoceles are congenital outpouchings of the larynx. Zenker’s diverticulum is a
dilatation in the pharyngeal wall due to failure of relaxation of the cricothyroid while swallowing.
References:
Sansur CA, Early S, Reibel J, Arlet V. Pharyngocutaneous fistula after anterior cervical spine
surgery. Eur Spine J. 2009 May;18(5):586-591.
Pubmed Web link: https://pubmed.ncbi.nlm.nih.gov/19330360/
Goz V, Qureshi S, Hecht AC. Arytenoid dislocation as a cause of prolonged hoarseness after
cervical discectomy and fusion. Global Spine J. 2013 Mar;3(1):47-50.
Pubmed Web link: https://pubmed.ncbi.nlm.nih.gov/24436851/
Epstein NE. A Review of Complication Rates for Anterior Cervical Diskectomy and Fusion
(ACDF) Surg Neurol Int. 2019 Jun 7;10:100.
Pubmed Web link: https://pubmed.ncbi.nlm.nih.gov/31528438/

65
Q

A 34-year-old woman has a 14-week history of severe, left lower extremity pain radiating in an S1distribution. Which of the following best describes differences in outcomes between tubular discectomy and open microdiscectomy?
Answers:
A. There is significant difference showing superiority of tubular microdiscectomy in short term follow-up compared to open microdiscectomy
B. There is no significant difference in relief of leg pain in short and long term follow-up when comparing tubular microdiscectomy vs open microdiscectomy
C. There is significant difference showing superiority of open microdiscectomy in long term follow-up compared to tubular microdiscectomy
D. There is significant difference showing superiority of open microdiscectomy in short term follow-up compared to tubular microdiscectomy
E. There is significant difference showing superiority of tubular microdiscectomy in long term follow-up compared to open microdiscectomy

A

There is no significant difference in relief of leg pain in short and long term follow-up when comparing tubular microdiscectomy vs open microdiscectomy

There is no significant difference in relief of leg pain in short and long term follow up when
comparing tubular microdiscectomy and open microdiscectomy. Baseline VAS was 6.9 and 7.2 in
microdiscectomies vs open discectomies, respectively1. There was no significant intergroup
difference in pain relief with either short term (2-3 months) follow up (VAS score 0.81, 95% CI
−4.71 to 6.32) or long-term (12-24 months) follow-up (VAS score 2.64, 95% CI −2.15 to 7.43).
There have been shown in some reports that tubular microdiscectomies may result in higher
incidence of incidental durotomy. Reoperation for recurrent herniation was more common in
patients randomized to the minimally invasive group (8.50% compared with 5.35% in patients
randomized to the open discectomy group), but this difference was not statistically significant (RR
1.56, 95% CI 0.92–2.66). Total complications did not differ significantly between the operations
(RR 1.50, 95% CI 0.97–2.33).
References:
Dasenbrock HH, Juraschek SP, Schultz LR, et al. The efficacy of minimally invasive discectomy
compared with open discectomy: a meta-analysis of prospective randomized controlled trials. J
Neurosurg Spine. 2012 May;16(5):452-62.
Pubmed Web link: https://www-ncbi-nlm-nih-gov.geihsl.idm.oclc.org/pmc/articles/PMC3618291/
Kamper SJ, Ostelo RW, Rubinstein SM, et al. Minimally invasive surgery for lumbar disc
herniation: a systematic review and meta-analysis. Eur Spine J. 2014 May;23(5):1021-43.2.
Pubmed Web link: https://link-springer-com.geihsl.idm.oclc.org/article
/10.1007%2Fs00586-013-3161-2

66
Q

A 68-year-old man is brought to the emergency department two days after being injured in an assault. He is intoxicated and obese. Clinical evaluation is insufficient to rule out cervical spine injury because of which of the following factors?
Answers:
A. Blunt trauma via assault
B. Emergency department setting
C. Length of time since injury
D. Obesity
E. Intoxication

A

Intoxication

Discussion:
The National Emergency X-Radiography Utilization Study (NEXUS) recommends the following
criteria for clinical clearance of adult (18 years or older) blunt cervical spine injuries: normal
neurological exam, no palpable midline tenderness, no painful distracting injuries, no altered level
of consciousness, and no alcohol. In published studies these criteria were able to identify almost
all clinically important injuries.
References:
Hoffman JR, Mower WR, Wolfson AB, et al. Validity of a set of clinical criteria to rule out injury to
the cervical spine in patients with blunt trauma. National Emergency X-Radiography Utilization
Study Group. N Engl J Med 2000;343:94-9
Pubmed Web link: https://pubmed.ncbi.nlm.nih.gov/10891516/
April RS, Lanfranchi R. Clinical criteria to rule out cervical-spine injury. N Engl J Med. 2000 Nov
2;343(18):1339. doi: 10.1056/NEJM200011023431813. PMID: 11183569.
Pubmed Web link: https://pubmed.ncbi.nlm.nih.gov/11183569